SMB Week 2

Lakukan tugas rumah & ujian kamu dengan baik sekarang menggunakan Quizwiz!

A researcher is investigating the structure of the sarcomere using skeletal muscle obtained from an experimental animal. He develops monoclonal antibodies directed against a specific skeletal muscle protein and finds that these antibodies disrupt the binding of actin to structural support elements within the sarcomere. Electron microscopy of the sarcomere is shown in the image below. ID=1734 Which of the following labeled regions do these antibodies most likely bind? A B C D E F

A sarcomere is composed of overlapping filaments of actin and myosin as well as structural and binding proteins (eg, titin, α-actinin). A single sarcomere is defined as the distance between two Z lines. Thin (actin) filaments are bound to structural proteins at the Z line. The unbound ends of the actin filaments project into the middle of the sarcomere, where they interact with thick (myosin) filaments during muscle contraction. (Choice B) The A band corresponds to myosin filaments in the sarcomere. In this region, there are myosin filaments overlapped with actin filaments as well as non-overlapped myosin filaments. The A band always remains the same length during muscle contraction. (Choice C) The M line lies at the center of the A band and is where myosin filaments anchor to structural elements in the center of the sarcomere. (Choice D) Myosin and actin filaments overlap at the segment of the sarcomere between the H and I bands. (Choices E and F) The I band is the region of the sarcomere in which actin does not overlap with myosin, and the H band is the region in which myosin does not overlap with actin. During muscle contraction, both H and I bands decrease in length. Educational objective: A single sarcomere is defined as the distance between two Z lines. Thin (actin) filaments in the I band are bound to structural proteins at the Z line, whereas thick (myosin) filaments in the A band are bound to structural proteins at the M line.

A 23-year-old man is being evaluated for myoclonic epilepsy of recent onset. The episodes are short-lived and triggered by startle. Physical examination reveals proximal muscle weakness. Gomori trichrome stain of a muscle biopsy specimen shows muscle fibers with a blotchy red appearance. No family history is available because the patient is adopted. What is the probability that this patient's one offspring will inherit the disease? 100% 75% 50% 25% 0%

Blotchy red muscle fibers on Gomori trichrome stain are characteristic of the mitochondrial myopathies. In these conditions, abnormal mitochondria accumulate under the sarcolemma of muscle fibers. The fibers have an irregular shape and size on cross section. For this reason, the mitochondrial myopathies are also known as the "red ragged fiber" diseases. Electron microscopy of the affected muscle reveals an increased number of enlarged, abnormally shaped mitochondria. The most important mitochondrial myopathies are myoclonic epilepsy with ragged red fibers (MERRF, described in this vignette), Leber optic neuropathy (blindness), and mitochondrial encephalopathy with stroke-like episodes and lactic acidosis (MELAS). Each of these diseases is the result of a mitochondrial gene mutation. The mitochondrial diseases are caused by mutations, deletions or duplications of mtDNA. Because sperm mitochondria do not pass into the ovum during fertilization, only maternal mitochondria are transmitted to the fetus. Mitochondrial diseases, therefore, have maternal inheritance. For this reason the male patient described in this vignette will not transmit the disease to his progeny. (Choice B) No pure single-gene inherited disease shows 75% transmission. There may be 75% transmission for a disease with variable penetrance. Individuals affected by a disease with autosomal dominant inheritance receive one copy of the abnormal gene from one parent. The chance that an affected person will transmit the disease to his/her child is 50% (Choice C) if the parent is heterozygous for the given disease. If the parent is homozygous, there is a 100% chance (Choice A) of transmission. (Choice D) Autosomal recessive diseases occur when the individual inherits two copies of the mutated gene (one from each parent). If both parents are carriers, there is a 25% chance that the child will be affected. Educational Objective: "Red ragged" muscle fibers are seen in mitochondrial diseases. Muscle fibers have this appearance because abnormal mitochondria accumulate under the sarcolemma. Mitochondrial diseases show maternal inheritance.

A 48-year-old previously healthy woman comes to the office due to progressively worsening muscle weakness for the past 2 months. The patient has difficulty with activities such as climbing stairs, getting up from chairs, and placing dishes in overhead cabinets. She also reports a 4.5-kg (10-lb) unintentional weight loss and occasional abdominal discomfort over the same interval. Physical examination shows weakness of the shoulder and hip girdle muscles. Other examination findings are shown in the image below. ID=11805 Further evaluation of this patient is most likely to reveal which of the following associated conditions? Chronic hepatitis C infection Ovarian adenocarcinoma Primary billiary cholangitis Tropheryma whippelii infection Type 2 diabetes mellitus

Dermatomyositis is a systemic autoimmune disease characterized by proximal muscle weakness resembling polymyositis, with additional inflammatory features involving the skin. Specific skin findings include the heliotrope rash in the periorbital area and cheeks and Gottron papules, which are raised erythematous plaques over the joints and bony prominences of the hands (as seen in this patient). Muscle biopsy is diagnostic and shows mononuclear perifascicular inflammation and atrophy (ie, occurring along the periphery of the fascicles). Dermatomyositis may occur alone or as a paraneoplastic syndrome of an underlying malignancy, most commonly due to underlying adenocarcinoma (eg, ovary, lung, pancreas). Symptoms may precede the diagnosis of malignancy but often parallel the course of the cancer. (Choice A) Hepatitis C infection is associated with porphyria cutanea tarda, which presents with skin fragility and a blistering rash in sun-exposed areas, and mixed cryoglobulinemic vasculitis, which presents with nonblanching palpable purpura. Muscle symptoms are not typical. (Choice C) Primary biliary cholangitis (previously primary biliary cirrhosis) is an autoimmune liver disease that can present with cholestasis, malabsorption, pruritus, and jaundice. Although the risk of this condition is increased in patients with autoimmune disease (eg, CREST syndrome, Sjögren syndrome), the association with dermatomyositis is not common. (Choice D) Tropheryma whippelii infection (Whipple disease) typically presents with arthritis, diarrhea, and fever. Patients may develop hyperpigmentation, but skin and muscle symptoms are otherwise not prominent. (Choice E) Type 2 diabetes is due to insulin resistance, which may also cause acanthosis nigricans, a condition characterized by velvety patches of brownish discoloration affecting the neck and skinfolds. Educational objective: Dermatomyositis is characterized by proximal muscle weakness resembling polymyositis, with additional inflammatory features involving the skin (heliotrope rash, Gottron papules). Both dermatomyositis and polymyositis may occur alone or as a paraneoplastic syndrome associated with an underlying adenocarcinoma (eg, ovary, lung, pancreas).

A 62-year-old woman comes to the office due to a 3-month history of progressive muscle weakness. The patient has been having difficulty with activities such as getting out of the car and carrying groceries from the store. She has also developed a rash on her face and hands. The patient has a history of well-controlled hypertension and does not use tobacco, alcohol, or illicit drugs. Vital signs are within normal limits. Physical examination shows weakness of shoulder abduction and hip flexion. Strength of the distal limb muscles and deep tendon reflexes are normal. Skin examination findings are shown in the exhibit. Which of the following is the most appropriate initial treatment for this patient's current condition? Acetylcholinesterase inhibitors Copper supplementation Riluzole therapy Systemic glucocorticoids Thyroid hormone replacement

Dermatomyositis is an autoimmune disorder characterized by: Cutaneous manifestations: Gottron papules (red or violaceous, flat-topped papules over joints and bony prominences, especially on the hands) and heliotrope rash (erythematous or violaceous edematous eruption on the upper eyelids and periorbital skin) Myopathy: Proximal muscle weakness (similar to polymyositis), elevated muscle enzymes (eg, creatine kinase, aldolase) Autoantibodies, including antinuclear antibodies (more sensitive) and anti-Jo-1 antibodies (more specific), are frequently positive. Muscle biopsy confirms the diagnosis and shows perifascicular inflammation and atrophy affecting a contiguous portion of the fascicle and surrounding blood vessels. Initial treatment includes systemic glucocorticoids (eg, prednisone). Dermatomyositis may occur alone or as a paraneoplastic syndrome associated with malignancy (especially adenocarcinoma); affected patients should be evaluated for an underlying occult malignancy. (Choice A) Acetylcholinesterase inhibitors (eg, physostigmine) are used in treatment of myasthenia gravis, which presents with fluctuating and fatigable weakness that primarily involves the ocular, bulbar, and respiratory muscles. The skin is not typically involved. (Choice B) Copper deficiency typically presents with muscle weakness, anemia, and fragile (kinky) hair. Skin hypopigmentation is also common, but rash does not occur. (Choice C) Riluzole is a glutamate inhibitor that is indicated for amyotrophic lateral sclerosis, a progressive neurodegenerative disorder that affects both upper and lower motor nerves. Although proximal muscle weakness may be seen (lower motor neuron dysfunction), upper motor neuron signs (eg, bulbar dysfunction, hyperreflexia) are expected and the disease is not associated with skin rash. (Choice E) Hypothyroid myopathy commonly presents with proximal muscle weakness. However, unlike polymyositis and dermatomyositis, patients usually also have prominent muscle pain and often have other hypothyroid features (eg, weight gain, cold intolerance). Educational objective: Dermatomyositis is characterized by proximal muscle weakness (similar to polymyositis) and dermal manifestations (eg, heliotrope rash, Gottron papules). Laboratory testing shows elevated muscle enzymes (eg, creatine kinase) and autoantibodies (eg, antinuclear, anti-Jo-1). Initial treatment includes systemic glucocorticoids and evaluation for potential underlying malignancy.

A 5-month-old boy is brought to the office due to poor feeding. His mother says that he has difficulty holding his head up while breastfeeding and his suckling seems weaker than usual. Weight is at the 5th percentile. Length and head circumference are tracking along the 25th percentile. Physical examination shows hepatomegaly and hypotonia in all 4 limbs. Cardiac auscultation shows a gallop rhythm, and chest x-ray reveals severe cardiomegaly. Muscle biopsy shows enlarged lysosomes containing periodic acid-Schiff (PAS)-positive material. Which of the following enzymes is most likely deficient in this patient? Acid alpha-glucosidase Galactokinase Glucose-6-phosphatase Glycogen debrancher enzyme Glycogen phosphorylase Pyruvate kinase

ID=1023 This patient most likely has glycogen storage disease type II (Pompe disease). This condition is caused by deficiency of acid alpha-glucosidase (alpha-1,4 glucosidase or acid maltase), an enzyme responsible for breaking down glycogen within the acidic environment of lysosomes. Although most glycogen is degraded in the cytoplasm, a small percentage is inadvertently engulfed by lysosomes, especially in cells containing high amounts of glycogen such as hepatocytes and myocytes. Deficiency of acid maltase results in pathologic accumulation of glycogen within liver and muscle lysosomes. Cardiac and skeletal muscle are particularly susceptible because the ballooning lysosomes interfere with contractile function. The classic form of the disease presents in early infancy with marked cardiomegaly, severe generalized hypotonia, macroglossia, and hepatomegaly. Blood glucose levels are normal, unlike with glycogen storage diseases that primarily affect the liver (eg, von Gierke disease). A key distinguishing feature is that muscle biopsy will show accumulation of glycogen in lysosomes. (Choice B) Galactokinase catalyzes the phosphorylation of galactose to galactose-1-phosphate in the first committed step of galactose catabolism. Galactokinase deficiency causes neonatal cataract formation due to accumulation of galactitol in the lens. (Choices C, D, and E) Other glycogen storage diseases are caused by deficiencies of glucose-6-phosphatase, glycogen phosphorylase, and glycogen debrancher enzyme (debranching enzyme). However, glycogen accumulation within lysosomal vacuoles is specific for acid alpha-glucosidase deficiency. (Choice F) Pyruvate kinase deficiency causes chronic hemolytic anemia, splenomegaly, and iron overload as a result of impaired erythrocyte survival. Educational objective: Acid maltase (alpha-glucosidase) deficiency presents in early infancy with cardiomegaly, macroglossia, and profound muscular hypotonia. Abnormal glycogen accumulation within lysosomal vesicles is seen on muscle biopsy.

A 78-year-old man was found to have a perihilar mass on screening CT scan. The patient's medical history is remarkable for chronic obstructive pulmonary disease, for which he takes albuterol inhalers as needed. He has smoked a pack of cigarettes daily for the last 50 years and does not drink alcohol or use illicit drugs. The patient is admitted for bronchoscopy and is premedicated with intramuscular atropine and becomes acutely restless, disoriented, and combative. Temperature is 38.1 C (100.5 F), blood pressure is 116/72 mm Hg, pulse is 110/min, and respirations are 15/min. Oxygen saturation is 99% on room air. On physical examination, his pupils are widely dilated and nonreactive to light. ECG shows sinus tachycardia. Which of the following agents will reverse all of this patient's signs and symptoms? Diazepam Edronphonium Haloperidol Neostigmine Physostigmine

ID=1564 Atropine is an anticholinergic medication that can be administered prior to bronchoscopy to decrease respiratory mucus secretions and promote bronchodilation. Anticholinergic drugs competitively inhibit the muscarinic acetylcholine receptor both centrally (leading to delirium, coma, and respiratory failure) and peripherally (see toxidrome in table). The elderly are at particularly high risk of developing anticholinergic toxicity, likely due to decreased renal and hepatic clearance. Cholinesterase inhibitors overcome this toxicity by inhibiting the degradation of acetylcholine, thereby increasing the concentration of acetylcholine at the synaptic cleft. Central nervous system (CNS) penetration and reversal of central symptoms are dependent on chemical structure: Tertiary amines (eg, physostigmine, galantamine, donepezil, rivastigmine) are lipophilic (nonpolar) and can easily cross the blood-brain barrier to reverse both central and peripheral symptoms. Quaternary amines (eg, neostigmine, edrophonium, pyridostigmine) are hydrophilic (polarized) and do not readily cross the blood-brain barrier. These drugs reverse peripheral symptoms only (Choices B and D). (Choices A and C) Both diazepam and haloperidol can be used for sedation or cases of severe agitation. Haloperidol is an antipsychotic that blocks dopamine receptors in the CNS, and diazepam is a long-acting benzodiazepine that positively modulates GABA-A activity. Neither medication reverses the peripheral anticholinergic manifestations (eg, mydriasis, tachycardia). Educational objective: Physostigmine is a cholinesterase inhibitor with a tertiary ammonium structure that can reverse both the central and peripheral nervous system symptoms of anticholinergic toxicity. Neostigmine, edrophonium, and pyridostigmine have a quaternary ammonium structure that limits central nervous system penetration.

A 31-year-old man comes to the office for a routine checkup. He has no significant medical problems and does not take any medications. The patient works as a fitness trainer and lifts weights recreationally. He has been consuming carbohydrate-rich food prior to his weightlifting sessions and claims that it increases muscle strength. A literature review shows that the rate of glycogenolysis within myocytes increases several hundredfold during active skeletal muscle contraction. Which of the following substances is most likely responsible for increasing the reaction rate during active contraction? ATP Ca2+ cAMP Glucose-6-phosphate Lactate

ID=1028 Glycogen is broken down by the enzyme glycogen phosphorylase, which is regulated through phosphorylation (active state) and dephosphorylation (inactive state). Phosphorylase kinase (PK) is the enzyme responsible for the phosphorylation of glycogen phosphorylase, whereas phosphoprotein phosphatase catalyzes its dephosphorylation. PK is regulated differently in liver than in muscles. Glycogen stored in the liver is used to maintain blood glucose levels during the fasting state, whereas glycogen in the muscles is used to provide energy for muscle contraction. In the liver, PK is activated primarily through the binding of epinephrine and glucagon to Gs protein-coupled receptors, which increases cAMP concentrations and causes phosphorylation of PK (via protein kinase A). Skeletal muscle lacks glucagon receptors, but muscle PK can still be phosphorylated in response to an epinephrine-induced increase in cAMP concentrations. However, increased intracellular calcium is a more powerful activator of muscle PK. Release of sarcoplasmic calcium stores following neuromuscular acetylcholine stimulation allows for synchronization of skeletal muscle contraction and glycogen breakdown, providing the energy necessary for anaerobic muscle contraction. (Choices A and D) Phosphorylated glycogen phosphorylase (active form) is allosterically inhibited by ATP and glucose-6-phosphate in both liver and muscle cells. Increased intracellular ATP levels help to decrease the rate of glycogenolysis upon cessation of active muscle contraction. (Choice C) Although increased cAMP stimulates muscle glycogen breakdown via the action of epinephrine on beta-1 adrenergic receptors, it is not responsible for synchronization of active muscle contraction and glycogen breakdown. (Choice E) Lactate is produced in tissues during anaerobic glycolysis, such as in muscles during strenuous exercise as a result of relatively hypoxic conditions. The lactate produced by the muscles can be converted to glucose in the liver via gluconeogenesis. Educational objective: Synchronization of glycogen degradation with skeletal muscle contraction occurs due to release of sarcoplasmic calcium following neuromuscular stimulation. Increased intracellular calcium causes activation of phosphorylase kinase, stimulating glycogen phosphorylase to increase glycogenolysis.

A 40-year-old woman is evaluated for neck pain that has gradually worsened over the past several months. The patient reports a pulling sensation in her neck that causes difficulty in keeping her head straight. She has had no trauma and has no other medical conditions. Physical examination shows a spastic and thickened sternocleidomastoid muscle causing head tilt. The patient is diagnosed with idiopathic torticollis. She is treated with injections of a bacterial product into the affected muscle, with significant relief of her symptoms. The substance administered to this patient is derived from bacteria that most likely demonstrate which of the following characteristics? Antigenically hypervariable pili Antiphagocytic capsule IgG-binding outer membrane protein Intracellular polyphosphate granules Subterminal spore formation Type III secretion system

ID=11458 missing ID=1401 Clostridium botulinum is a gram-positive, anaerobic bacillus that synthesizes botulinum toxin intracellularly and releases it by autolysis. Botulinum toxin prevents the presynaptic release of acetylcholine from the nerve terminal at the neuromuscular junction. This prevents both nicotinic and muscarinic activation, leading to skeletal muscle weakness and reduced glandular secretions, respectively. Botulism is a result of toxin ingestion and is characterized by cranial nerve deficits and descending symmetric paralysis. Spore formation, which occurs in the subterminal location (between the terminal end and center of the bacteria), allows the bacteria to survive adverse conditions (eg, heat, high-oxygen environment). Focal dystonia is a localized, uncontrollable muscle contraction that can lead to pain and physical deformity. Torticollis is a common manifestation of dystonia and is due to involvement of the sternocleidomastoid muscle. Injection of botulinum toxin type B into the dystonic muscle results in muscular relaxation and relief of symptoms. Botulinum toxin can also be used to treat the muscle spasms of multiple sclerosis and Parkinson disease, to relax the lower esophageal sphincter in achalasia, and to reduce the appearance of facial wrinkles. The localized anticholinergic effect also makes it useful in the treatment of hyperhydrosis. The effects are temporary (approximately 3 months), due to the regeneration of the nerve terminal with time. (Choice A) Hypervariable pili are characteristic of Neisseria meningitidis and Neisseria gonorrhoeae; antigenic heterogeneity aids in immune evasion and allows for repeat infections. (Choice B) The antiphagocytic capsule is a primary virulence factor for Streptococcus pneumoniae, Haemophilus influenzae, Klebsiella pneumonia, and Neisseria meningitidis. (Choice C) Staphylococcus aureus has an IgG-binding outer membrane protein—the protein A virulence factor—which binds the Fc portions of IgG, thereby preventing opsonization, phagocytosis, and complement fixation. (Choice D) Intracellular polyphosphate granules are seen on methylene blue staining and are an identifying characteristic of Corynebacterium diphtheriae. (Choice F) Type III secretion systems allow bacteria to inject proteins directly into host cells to avoid invoking an immune response. They are seen in many gram-negative bacteria, including Shigella, Salmonella, Pseudomonas, Yersinia, and enteropathogenic Escherichia coli. Educational objective: Clostridium botulinum is a spore-forming, gram-positive, anaerobic bacillus that synthesizes botulinum toxin, which prevents the presynaptic release of acetylcholine from the nerve terminal at the neuromuscular junction. Botulinum toxin can be used to treat focal dystonia and other disorders of abnormal muscle contraction.

A 13-year-old boy comes to the office for evaluation of worsening scoliosis. He lost the ability to walk a year ago and has since been wheelchair bound. He has significant back pain and difficulty sitting in his wheelchair for extended periods. The patient also feels "too weak" to cough or take a deep breath. Review of medical records shows genetic studies confirming a mutation of the dystrophin gene on X chromosome p21. Histopathology of the patient's calf most likely indicates which of the following findings? Fibrofatty muscle replacement Hypertrophic muscle fibers Multiple lipid droplets within muscle fibers Prominent inflammatory infiltrate Ragged red fibers

ID=1266 Deletion of the dystrophin gene causes Duchenne muscle dystrophy (DMD). The condition is X-linked recessive and therefore affects primarily boys. Dystrophin is a structural protein of muscle fibers, and its absence causes muscle fiber destruction (myonecrosis). Variation in muscle fiber shape and size, regenerating fibers, and increased amounts of connective tissue are seen on light microscopy. Disease onset is age 2-5. Muscles of the proximal lower extremities, back, and pelvic and shoulder girdles are affected first. Symptoms of DMD include the following: Ambulation difficulties: Clumsy, slow, waddling gait; cannot keep up with peers Gower sign: Progressive weakness in proximal musculature, resulting in use of the hands to support weight on standing (as shown in the image above) Calf pseudohypertrophy: Calf muscles hypertrophy initially in response to proximal muscle weakness and are later replaced by fat and connective tissue Asymmetric weakening of the paraspinal muscles, leading to kyphoscoliosis Most patients with DMD are wheelchair bound by age 12. Scoliosis progresses rapidly due to muscle imbalance and body positional changes. Worsening scoliosis is complicated by restrictive pulmonary function (decreased vital capacity and total lung capacity). (Choice B) Hypertrophy of muscle fibers may occur in the early stages of DMD. However, fibrofatty change of the enlarged muscles would be expected in a patient at this advanced stage of the disease. (Choice C) Lipid accumulation within muscle fibers is seen in lipid myopathies such as carnitine palmitoyltransferase deficiency. (Choice D) Prominent inflammatory infiltrate is a characteristic histologic feature of polymyositis and dermatomyositis. These conditions cause proximal muscle weakness without distal pseudohypertrophy. (Choice E) Ragged red fibers refer to muscle fibers with irregular contours and a blotchy red appearance. They are seen in mitochondrial myopathies. Educational objective: Duchenne muscle dystrophy manifests with proximal muscle weakness and atrophy. True hypertrophy of the distal muscle is noted early in the disease as distal muscles compensate for weak proximal ones. Muscle fibers of the distal extremities are later replaced by fat and connective tissue (pseudohypertrophy).

A 21-year-old man comes to the clinic due to weakness and gait disturbance. For the past few months, he has noticed difficulty opening the lids on jars, along with trouble releasing his grip afterward. The patient has also tripped and fallen several times, particularly when walking up stairs. He has no known chronic medical conditions and takes no medications. Vital signs are normal. The patient has male pattern baldness. Cardiopulmonary examination is unremarkable. The abdomen is soft with no organomegaly. A muscle biopsy shows muscle atrophy involving mostly type 1 fibers. This patient most likely has a form of which of the following neuromuscular diseases? Inflammatory myopathy Ion channel myopathy Mitochondrial myopathy Motor neuron disease Muscular dystrophy Neuromuscular junction disease

ID=1268 This patient has distal muscle weakness (eg, difficulty opening jar lids) with delayed muscle relaxation after contraction (myotonia), key features of myotonic dystrophy (DM). DM is an autosomal dominant muscular dystrophy caused by a trinucleotide repeat expansion in the DMPK gene, which is then transcribed into mutant mRNA. The mutant mRNA is untranslatable and toxic to the expression of other genes, particularly those related to skeletal muscle chloride channels, resulting in myotonia. Grip myotonia is classic and characterized by impaired muscle relaxation after voluntary release of a contracted hand. The toxic mRNA also affects T tubules, which usually facilitate synchronized release of calcium ions for organized muscle contraction, resulting in weakness due to impaired excitation-contraction coupling. Weakness classically involves the face (eg, long, narrow face with sagging) and distal skeletal muscles. This patient's recent falls are likely due to footdrop, but quadriceps involvement can occasionally occur and may be contributing to his difficulty walking up stairs. Muscular atrophy in DM preferentially affects the type I (slow-twitch) fibers, as seen on this patient's muscle biopsy. Other common features of DM include cataracts, insulin resistance, and early-onset frontal balding. (Choice A) Inflammatory myopathies (eg, dermatomyositis, polymyositis) present with symmetric proximal muscle weakness without myotonia. In addition, inflammatory cell infiltration is seen on muscle biopsy. (Choice B) Certain ion channel myopathies can manifest with myotonia, but those that do so are associated with muscle hypertrophy (not atrophy) due to muscle overactivity. In contrast, the genetic defects causing muscular dystrophy result in progressive muscle damage that leads to atrophy of muscle fibers. (Choice C) Mitochondrial myopathies are characterized by varying degrees of muscle weakness, typically presenting in infancy or early childhood with multisystem involvement (eg, seizures). In addition, ragged red fibers (due to bright red mitochondrial bundles) are classically seen on muscle biopsy. (Choice D) Motor neuron disease (eg, amyotrophic lateral sclerosis) is characterized by progressive nerve degeneration, resulting in skeletal muscle weakness. Muscle wasting and a weak grip are common, but impaired muscle relaxation does not occur. (Choice F) Myasthenia gravis, caused by postsynaptic acetylcholine receptor antibodies, is the most common neuromuscular junction disorder and presents with proximal muscle weakness. However, the weakness is fluctuating and fatigable, and hand weakness and relaxation impairment would not be expected. Educational objective: Myotonic dystrophy is characterized by muscle weakness and myotonia (impaired muscle relaxation) with muscle atrophy, particularly of the type I (slow-twitch) muscle fibers. Pathophysiology involves a trinucleotide repeat expansion causing mutant RNA that disrupts muscle contraction and relaxation.

A 50-year-old man is brought to the emergency department after developing blurred vision and confusion. Earlier today, he was clearing and burning foliage and weeds on his property. His medical history is insignificant, and he does not take any medications. The patient has no history of illicit drug use. Temperature is 38.9 C (102 F), blood pressure is 110/76 mm Hg, pulse is 120/min, and respirations are 16/min. He is disoriented to time, place, and person. Physical examination shows flushed skin and dry oral mucosa. Both pupils are dilated and nonreactive to light and accommodation. Bowel sounds are decreased. Which of the following drugs can potentially reverse this patient's condition? Atropine Benztropine Diazepam Haloperidol Metoprolol Morphine Naloxone Physostigmine Thiamine

ID=1321 This patient with blurred vision, fever, altered mental status, flushed skin, and dry mucous membranes has anticholinergic toxicity, likely from encountering jimsonweed (Datura stramonium) while working in his yard. Jimsonweed contains large concentrations of the anticholinergic compounds atropine, scopolamine, and hyoscyamine. These agents competitively inhibit acetylcholine at the muscarinic acetylcholine receptor, leading to the classic toxidrome summarized above. Anticholinergic toxicity can be counteracted by increasing the concentration of acetylcholine in the synaptic cleft. Physostigmine, a cholinesterase inhibitor, increases acetylcholine levels by preventing its degradation by cholinesterase. (Choice A) Organophosphates are cholinesterase inhibitors that are commonly used as pesticides. Atropine can be used to counteract the effects of excess muscarinic stimulation (eg, increased salivation, miosis, bronchospasm, bradycardia). However, patients remain at risk of paralysis due to nicotinic overactivation, and so also require treatment with pralidoxime, a cholinesterase reactivating agent. (Choices B and D) Benztropine is a centrally acting anticholinergic medication used for the treatment of idiopathic and drug-induced Parkinson disease. Haloperidol is a neuroleptic drug that blocks dopamine receptors in the central nervous system and is used to treat psychosis. These drugs have anticholinergic effects that would worsen this patient's symptoms. (Choice C) Diazepam, a long-acting benzodiazepine, positively modulates GABAA action by increasing the frequency of chloride channel opening. It is used to treat seizures associated with atropine poisoning but does not affect muscarinic cholinergic receptors. (Choice E) Metoprolol is a selective β1-adrenergic receptor antagonist. It is used to treat angina, acute coronary syndromes, heart failure, hypertension, and arrhythmias. (Choices F and G) Morphine is an opioid pain medication that exerts its effects by stimulating opioid receptors in the brain. Naloxone is a competitive antagonist of these receptors used for opioid overdose. Morphine has some anticholinergic effects and may worsen symptoms whereas naloxone does not affect cholinergic receptors. (Choice I) Thiamine (B1) is a cofactor for multiple enzymes used in glucose metabolism. Patients who are chronically deficient (eg, due to alcohol use disorder or malnutrition) can develop Wernicke-Korsakoff syndrome (ataxia, ophthalmoplegia, confusion). Educational objective: Anticholinergic agents (eg, atropine, scopolamine) competitively inhibit acetylcholine at the muscarinic acetylcholine receptor. The effects can be memorized with the mnemonic, "Blind as a bat, mad as a hatter, red as a beet, hot as a hare, dry as a bone, full as a flask, and fast as a fiddle," and can be reversed by cholinesterase inhibitors (physostigmine).

An 18-year-old man, who was recently started on risperidone therapy for schizophrenia, comes to the office due to involuntary deviation of his head. On examination, the patient's head is tilted toward the right side and the chin is rotated to the left. There is palpable tightening of the right sternocleidomastoid muscle. It is determined that the patient is experiencing a medication-induced dystonic reaction due to motor neuron hyperactivity. The persistent myocyte stimulation causes a substance to be released from the sarcoplasmic reticulum. This substance most likely binds to which of the following proteins to cause this patient's symptoms? Actin Myosin Tropomyosin Troponin Protein kinase A

ID=1558 The sarcoplasmic reticulum (SR) is a modified endoplasmic reticulum within skeletal muscle cells. The SR forms a network of tubules with terminal cisterns that are in close contact with the T tubules (cytoplasmic membrane invaginations), allowing the membrane depolarization signal to reach the SR. The Ca2+-ATPase pump in the SR membrane actively sequesters calcium to keep intracellular concentrations low. Cell membrane depolarization causes calcium release from the SR into the cytoplasm via the SR ryanodine receptor. Actin filaments, troponin complex (troponins C, T, and I), and tropomyosin form the thin filaments of muscle fibers. In the resting state, tropomyosin covers the myosin binding sites on the actin filaments. On release from the SR, calcium binds to troponin C on the thin filaments (Choices A and C). This induces a conformation change in the troponin complex, causing it to displace tropomyosin and expose the myosin binding sites on the actin filaments. Myosin makes up the thick filaments of skeletal muscle. ATP is bound by the myosin head and cleaved to form ADP and inorganic phosphate ion, which are retained at the myosin head. When the myosin head binds to an actin filament, a conformational change causes the myosin to pull the actin filament, leading to muscle contraction and ADP release. A new ATP molecule then is bound to the myosin head, causing release of the actin filament. The cycle then repeats until calcium is displaced from troponin C and the myosin binding sites are again covered (Choice B). The pulse of elevated intracellular calcium lasts only a fraction of a second before the calcium is resequestered within the SR. (Choice E) Protein kinase A is activated by high intracellular concentrations of cAMP and is involved in the second messenger pathways of numerous hormones, including glucagon, beta adrenergic receptors, parathyroid hormone and others. Educational objective: During skeletal muscle contraction, calcium is released from the sarcoplasmic reticulum and binds troponin C, thereby allowing the binding of actin to myosin.

As part of an experiment, radiolabeled ATP is injected into skeletal muscle. During muscle contraction, the labeled ATP is observed to attach to the sarcomere. This attachment causes immediate: Calcium binding to troponin C Tropomyosin displacement from the groove on the actin molecule Myosin head detachment from the actin filament Cross-bridge formation Myosin light chain phosphorylation by a specific enzyme

ID=1665 According to the model posited by Rayment et al., the role of ATP in skeletal and cardiac muscle contraction may be to release the myosin head from its actin binding site and then to energize a conformational change that resets the myosin head to "contract" again the next time it binds to actin. (Choices A and B) Calcium binding to troponin C shifts tropomyosin away from the myosin binding site on actin. These steps do not depend directly on ATP. (Choice D) If ATP is not available, the cross-bridge between myosin and actin will persist (rigor mortis). (Choice E) Myosin light chain kinase phosphorylates the myosin light chain, activating myosin to bind actin filaments in smooth muscle cells. There is no myosin head phosphorylation by a specific kinase in skeletal muscle contraction. Educational Objective: During the skeletal muscle contraction cycle, ATP binding to myosin causes release of the myosin head from its binding site on the actin filament.

A muscle biopsy obtained from a healthy volunteer consists mainly of myoglobin-rich, glycogen-poor fibers with many mitochondria. Which of the following is the most likely biopsy site? Biceps brachii Deltoid Latissimus dorsi Paraspinal Pectoralis major

ID=1858 The body's muscles are composed of 2 main fiber types: slow twitch (Type I) and fast twitch (Type II). Type I fibers perform actions requiring low-level sustained force (eg, postural maintenance) and function primarily via aerobic metabolism, meaning they have high myoglobin (oxygen storage) and mitochondrial (aerobic respiration) concentrations. The soleus muscle of the lower leg and paraspinal muscles of the back are postural muscles predominantly composed of Type I fibers. Type II fibers are specialized for generating rapid forceful movements and they fatigue quickly. These fibers derive ATP energy mainly via glycogenolysis and subsequent anaerobic glycolysis. Muscles that maneuver the upper extremities (Choices A, B, C, and E) are primarily geared toward rapid forceful movements and are more likely to have a higher percentage of Type II than Type I muscle fibers. Educational objective: Postural skeletal muscles such as the soleus and paraspinal muscles contain predominantly Type I, slow twitch muscle fibers that derive ATP primarily via oxidative (aerobic) metabolism.

A 59-year-old man comes to the office due to progressive weakness for the past several weeks. The patient has had difficulty getting out of bed and climbing stairs in the morning, which improves gradually as he "warms up the muscles" with continued use. He has also had to drink water more frequently due to dry mouth and has had episodic double vision. The patient also reports difficulty achieving erections despite having a good libido. Neurologic examination shows decreased strength of the hip flexors and diminished knee reflexes. Repeat examination after lower extremity isometric exercise shows of the muscle strength and deep tendon reflexes. Which of the following conditions is most closely associated with this patient's current disease process? Campylobacter jejuni enteritis Gastric adenocarcinoma Giant cell arteritis Medullary thyroid cancer Small cell lung cancer Thymic neoplasia

ID=1900 This patient with proximal muscle weakness and decreased deep tendon reflexes that improve with exercise has Lambert-Eaton myasthenic syndrome (LEMS). LEMS is an immune-mediated disorder of the neuromuscular junction characterized by autoantibodies directed at the presynaptic voltage-gated calcium channel, resulting in decreased acetylcholine release and muscle weakness. LEMS is strongly associated with small cell lung cancer, likely due to the immune recognition of voltage-gated calcium channels that are present on the malignant cells. Patients typically develop progressive proximal muscle weakness that manifests in the form of gait alteration and difficulty arising from a chair or climbing stairs. Autonomic symptoms, such as dry mouth or impotence, are common. Cranial nerve (particularly ptosis) and respiratory muscle involvement may occur but typically manifest later in the disease course. Patients may experience postexercise facilitation, in which muscle strength and deep tendon reflexes improve with exercise or repetitive movements (due to accumulation of calcium within the axon terminal). This effect is also seen in repetitive nerve stimulation studies. (Choice A) Guillain-Barré syndrome is an acute postinfectious polyneuropathy often associated with Campylobacter jejuni infections. It is characterized by rapidly progressive, ascending paralysis that does not improve with repetitive movement. (Choice B) Paraneoplastic syndromes associated with gastric adenocarcinoma include abrupt onset of numerous seborrheic keratoses (Leser-Trélat sign) and acanthosis nigricans. (Choice C) Giant cell arteritis is characterized by headache, visual changes, and jaw claudication in older adults. It is not associated with LEMS. (Choice D) Medullary thyroid cancer may be associated with multiple endocrine neoplasia (MEN) type 2 (both A and B variants). MEN 2A is associated with pheochromocytoma and hyperparathyroidism, whereas MEN 2B is associated with pheochromocytoma and mucosal neuromas. (Choice F) Thymic neoplasia or thymomas are commonly associated with myasthenia gravis (MG), another autoimmune neuromuscular condition. MG is often confused with LEMS; however, in MG facial, periocular, and bulbar weakness typically presents before extremity weakness. In addition, the weakness worsens with exercise, deep tendon reflexes typically remain intact, and autonomic dysfunction is rare. Educational objective: Lambert-Eaton myasthenic syndrome (LEMS) is a neuromuscular disorder characterized by autoantibodies against presynaptic voltage-gated calcium channels. It causes progressive proximal muscle weakness and decreased deep tendon reflexes that improve with exercise (postexercise facilitation); cranial nerve involvement and autonomic symptoms may also occur. LEMS is strongly associated with small cell lung cancer.

A 44-year-old man with progressive dyspnea is diagnosed with dilated cardiomyopathy. Despite optimal medical therapy, he continues to have symptoms and disease progression is noted. He undergoes cardiac transplantation after a suitable donor becomes available. Permission is obtained from the patient to study his diseased heart for intracellular calcium regulation. Microelectrodes placed into cardiac muscle cells detect a rapid decrease in cytoplasmic calcium level immediately preceding relaxation. Which of the following proteins is most likely responsible for the observed change in electrolyte levels? Calmodulin Na+/Ca2+ exchanger Ryanodine receptors Troponin C Voltage-dependent calcium channels

ID=1931 Intracellular calcium regulation plays an important role in excitation-contraction coupling. Voltage-dependent calcium channels (L-type) are activated during phase 2 of cardiac action potential (depolarization) and permit influx of calcium into the cardiac myocytes (Choice E). This initial calcium influx is sensed by the ryanodine receptors in the sarcoplasmic reticulum, which triggers further release of Ca2+ (calcium-induced calcium release) into the cytoplasm, thereby increasing intracellular calcium concentration 100-fold (Choice C). The calcium released from the sarcoplasmic reticulum diffuses through the myofilament network and binds to troponin C (Choice D). Tropomyosin is then moved out of the way so that actin and myosin can interact, leading to muscle contraction. The final stage of excitation-contraction coupling is myocyte relaxation, which occurs subsequent to calcium efflux from the cytoplasm. Intracellular calcium is removed primarily via an Na+/Ca2+ exchange pump (NCX) and sarcoplasmic reticulum Ca2+-ATPase pump (SERCA). NCX uses the large extracellular Na+ concentration gradient to help pump Ca2+ out of the cell and, in the process, removes one intracellular Ca2+ in exchange for 3 extracellular Na+ ions. In contrast, SERCA is a Ca2+-ATPase pump that actively transfers Ca2+ from the cytosol to the lumen of sarcoplasmic reticulum at the expense of ATP hydrolysis. (Choice A) Calmodulin (calcium modulated protein) is a calcium-binding messenger protein that is present in all cells and mediates many of the regulatory effects of Ca2+. Calmodulin is important for excitation-contraction coupling in smooth muscle cells, which lack troponin, unlike cardiac and skeletal muscles. In cardiac muscle, calmodulin is not directly involved in excitation-contraction coupling; rather, it helps regulate intracellular calcium activity and transcription factor signaling. Educational objective: Calcium efflux from cardiac cells prior to relaxation is primarily mediated via an Na+/Ca2+ exchange pump and sarcoplasmic reticulum Ca2+-ATPase pump.

Item 2 of 2 The patient is started on appropriate treatment and reports significant symptom improvement. However, she now has abdominal cramping, nausea, sweating, and diarrhea. Which of the following agents can be used to effectively control these new symptoms? Fluoxetine Glycopyrrolate Pilocarpine Prazosin Propranolol

ID=2062 The treatment of myasthenia gravis typically involves the use of a cholinesterase inhibitor, an immunosuppressive agent, and possible thymectomy. This patient is most likely receiving treatment with a cholinesterase inhibitor, such as pyridostigmine, which inhibits the degradation of acetylcholine in the neuromuscular junction. This improves skeletal muscle weakness but can cause muscarinic overstimulation of the smooth muscles and excessive glandular secretions (eg, diarrhea, diaphoresis, abdominal cramping, emesis). Selective muscarinic antagonists (eg, glycopyrrolate, hyoscyamine, propantheline) can be used to reduce the side effects of cholinesterase inhibitors in sites where acetylcholine action is mediated by muscarinic receptors (ie, gastrointestinal tract). Because of their selectivity, these drugs improve side effects without affecting the action of cholinesterase inhibitors on skeletal muscle, which uses nicotinic receptors. (Choice A) Fluoxetine is a selective serotonin reuptake inhibitor used primarily in the treatment of depression. It acts in a fashion similar to the cholinesterase inhibitors in that it prolongs the activity of a neurotransmitter on its target tissue. However, it is otherwise unrelated to cholinesterase inhibitors or their effects. (Choice C) Pilocarpine is a nonselective muscarinic receptor agonist. Use of this agent would worsen this patient's side effects. (Choices D and E) Prazosin is an alpha-1 adrenergic antagonist, and propranolol is a beta adrenergic antagonist. Use of either agent does not prevent the side effects of a cholinesterase inhibitor. Educational objective: The treatment of myasthenia gravis involves the use of a cholinesterase inhibitor, immunosuppressants, and possible thymectomy. Cholinesterase inhibitors may cause adverse effects related to muscarinic overstimulation, which can be ameliorated by the use of an antimuscarinic agent such as glycopyrrolate, hyoscyamine, or propantheline.

A 47-year-old man comes to the office due to worsening muscle weakness and dark urine for the past several days. Two weeks ago, the patient was evaluated for dyspepsia and found to have Helicobacter pylori infection; he was started on treatment and has had partial symptom improvement. Medical history includes hypertension and hyperlipidemia, for which he takes amlodipine and simvastatin. Temperature is 37.1 C (98.8 F), blood pressure is 130/80 mm Hg, and pulse is 74/min. Physical examination shows diffuse muscle tenderness. Laboratory evaluation reveals elevated blood urea nitrogen and serum creatinine levels. Urine microscopy shows no red or white blood cells. Which of the following medications most likely precipitated this patient's current condition? Amoxicillin Bismuth subsalicylate Calcium carbonate Clarithromycin Metronidazole

ID=2770 missing ID=161 This patient has muscle tenderness and weakness. The dark urine suggests myoglobinuria due to myocyte necrosis. In the context of ongoing statin therapy for hyperlipidemia, this presentation likely represents statin myopathy. Statin myopathy is due, likely at least in part, to decreased myocyte production of coenzyme Q10 (ubiquinone). Serum muscle breakdown markers (eg, creatine kinase) are often elevated, and severe cases may lead to rhabdomyolysis with subsequent acute kidney injury (eg, elevated BUN and creatinine). Statin myopathy is most common in the initial weeks or months of therapy. However, it can occasionally occur later, and can be acutely triggered by medications that increase circulating statin levels. This patient's myopathy is most likely related to the initiation of clarithromycin as part of a multidrug regimen for Helicobacter pylori. Statins, primarily simvastatin, lovastatin, and atorvastatin, are metabolized by CYP3A4. This enzyme can be inhibited by macrolide antibiotics (eg, erythromycin, clarithromycin), leading to increased statin drug levels and subsequent statin myopathy. (Azithromycin does not significantly inhibit CYP3A4.) Other significant CYP3A4 inhibitors that can induce statin myopathy include ketoconazole, non-dihydropyridine calcium channel blockers (eg, diltiazem), amiodarone, and protease inhibitors (eg, ritonavir). Because pravastatin is not primarily metabolized by CYP3A4, patients who must take a CYP3A4 inhibitor may benefit from switching to this drug. (Choice A) Amoxicillin does not inhibit CYP3A4 or increase circulating simvastatin levels. Penicillin antibiotics generally have only minor effects on the cytochrome P-450 system. (Choice B) Bismuth subsalicylate can cause black stools due to the formation of bismuth sulfate in the gastrointestinal tract. However, this medication does not cause myopathy or myoglobinuria. Excessive intake of bismuth can cause motor weakness, but this is due primarily to neural rather than muscle toxicity. (Choice C) Polyvalent cations (eg, calcium, iron) can form insoluble chelate complexes with certain antibiotics (eg, tetracyclines, fluoroquinolones) and other drugs (eg, levothyroxine, levodopa), leading to decreased drug absorption. Simvastatin does not form chelate complexes, and decreased absorption would lower, not increase, the risk for myopathy. (Choice E) Both metronidazole and simvastatin carry a small risk for peripheral neuropathy. In some patients, the effect can be additive and lead to numbness, pain, and paresthesia in the hands and feet. However, this neuropathy would not cause muscle tenderness or myoglobinuria. Educational objective: Statins, primarily simvastatin, lovastatin, and atorvastatin, are metabolized by CYP3A4. Drugs that inhibit this enzyme (eg, macrolide antibiotics, ketoconazole, non-dihydropyridine calcium channel blockers, amiodarone) can cause increased statin drug levels and lead to statin myopathy.

A 61-year-old woman comes to the clinic due to vision problems for the last 6 months. The patient says her eyes get tired easily, especially in the late afternoon and evening. She has smoked a pack of cigarettes daily for the past 35 years and does not use alcohol or illicit drugs. On physical examination, there is weakness of the extraocular muscles. Pupillary reflex, deep tendon reflexes, and sensation are normal. A contrast-enhanced CT scan of the chest is shown in the exhibit. Which of the following is the most likely mechanism underlying this patient's symptoms? Antibodies against a neurotransmitter receptor Antibodies against a presynaptic calcium channel CD8+ lymphocyte-mediated nerve fiber demyelination CD8+ lymphocyte-mediated skeletal muscle damage Vascular deposition of circulating immune complexes

ID=749 This patient's presentation is consistent with myasthenia gravis (MG), an autoimmune disease caused by autoantibodies directed against nicotinic acetylcholine receptors on the postsynaptic membrane of the neuromuscular junction. Acetylcholine binding normally opens these ligand-gated sodium channels, and the subsequent influx of sodium ions raises endplate potential. To produce a self-propagating muscle action potential, there must be enough activated acetylcholine receptors to allow the endplate potential to reach threshold. In MG, antibody binding leads to receptor degradation and complement-mediated damage of the postsynaptic membrane, preventing action potentials from being triggered. Patients commonly have fluctuating weakness that worsens throughout the day and most often involves the extraocular (eg, ptosis, diplopia) and bulbar (eg, fatigable chewing, dysphagia) muscles. Ptosis can increase when the patient sustains an upward gaze, and electrophysiologic studies may demonstrate decreasing force of muscle contraction with repetitive stimulation. Most patients have thymic abnormalities (eg, thymoma, thymic hyperplasia) that may appear as an anterior mediastinal mass on chest imaging. (Choice B) Lambert-Eaton syndrome is caused by autoantibodies directed against the presynaptic calcium channel, which causes reduced acetylcholine release. Patients usually have proximal extremity weakness that improves with muscle use, as opposed to the fatigable weakness of extraocular and bulbar muscles common in MG. In addition, hyporeflexia (vs normal reflexes) is typically present. (Choice C) Guillain-Barré syndrome is caused by segmental demyelination of peripheral nerves by infiltrating T lymphocytes (CD8+) and autoantibody formation. Patients with this acute inflammatory polyneuropathy can have ocular muscle weakness; however, symmetric ascending muscle weakness is classic, and symptoms typically progress rapidly over hours to days (vs months). (Choice D) Polymyositis is an autoimmune disease that occurs due to CD8+ lymphocyte-mediated skeletal muscle damage. It usually presents with symmetric proximal muscle weakness (eg, hip flexors) rather than extraocular muscle weakness. (Choice E) Vascular deposition of circulating immune complexes—as may occur in connective tissue disorders such as systemic lupus erythematosus and rheumatoid arthritis—can cause inflammation of blood vessel walls (vasculitis) and a wide range of symptoms. Muscle weakness can occur but is unlikely to be isolated to extraocular muscles or fluctuate over the course of the day. Educational objective: Myasthenia gravis is caused by circulating antibodies directed against acetylcholine receptors of the neuromuscular junction. Autoantibody binding leads to receptor degradation, producing fluctuating weakness that worsens throughout the day and most commonly affects the extraocular muscles (eg, ptosis, diplopia). Most patients have thymic abnormalities (eg, thymoma).

A 32-year-old woman complains of weakness in her hands and "heaviness" in her eyelids at the end of each day. Chest imaging shows an anterior mediastinal mass. The organ from which this mass most likely originated shares its embryologic origin with: Thyroid gland Superior parathyroid glands Inferior parathyroid glands Larynx Palatine tonsils

ID=756 Myasthenia gravis causes muscle weakness, with the extraocular muscles most commonly affected. Patients often experience ptosis and diplopia. The muscle weakness worsens with activity, and patients often note that their symptoms are worse at the end of the day. The majority of patients with myasthenia gravis are found to have a thymoma or thymic hyperplasia. The thymus is derived from the third pharyngeal pouch, as are the inferior parathyroid glands. The table below lists the derivatives of the pharyngeal pouches, grooves and membranes: (Choice A) The floor of the foregut gives rise to the thyroid diverticulum. The thyroid diverticulum migrates caudally to the neck, but remains attached to the floor of the mouth by the thyroglossal duct. (Choice B) The superior parathyroid glands and ultimobranchial body are formed from the 4th pharyngeal pouch. (Choice D) Most of the laryngeal cartilages develop from the 4th and 6th pharyngeal arches. (Choice E) The palatine tonsils are derived from the second pharyngeal pouches. Educational Objective: Myasthenia gravis is associated with abnormalities of the thymus (e.g. thymoma, thymic hyperplasia). The thymus and inferior parathyroid glands arise from the 3rd pharyngeal pouch.

Physiologists are studying the biomolecular mechanisms underlying skeletal muscle contraction. They have been analyzing muscle fibers obtained from knockout mice to determine how different cellular substances influence muscular contraction. Striated muscle fibers obtained from a specific mouse embryo fail to contract in response to a substance normally released from the sarcoplasmic reticulum. A deficiency of which of the following would best explain this finding? Acetylcholine Epinephrine Calmodulin Myosin light-chain kinase Troponin

ID=7592 The contractile mechanism in skeletal muscle depends on proteins (myosin II, actin, tropomyosin, and troponin) as well as calcium ions. The thick filaments in skeletal muscle are comprised of myosin molecules, with the heads of the myosin molecules forming cross-links with actin during muscular contraction. Two actin chains comprise the thin filaments in skeletal muscle. Tropomyosin molecules sit in the groove between the two actin chains, covering the myosin binding sites on actin when the muscle is at rest. Troponin molecules are small globular proteins situated alongside the tropomyosin molecules. Troponin is composed of three subunits: troponin T, troponin I, and troponin C. Troponin T binds the other troponin components to tropomyosin, troponin I binds the troponin-tropomyosin complex to actin, and troponin C contains the binding sites for Ca2+. During excitation-contraction coupling, Ca2+ is released from the sarcoplasmic reticulum. When Ca2+ binds troponin C, tropomyosin shifts to expose the actin binding sites for myosin, allowing contraction to occur. (Choice A) Acetylcholine is the neurotransmitter that initiates muscle contraction in response to motor neuron stimulation. Acetylcholine release from the motor neuron opens post-synaptic ligand-gated ion channels, resulting in depolarization of the muscle cell. Depolarization then causes release of Ca2+ from the sarcoplasmic reticulum. (Choice B) Epinephrine is a catecholamine that is not directly involved in skeletal muscle contraction. However, epinephrine stimulates β2-adrenergic receptors to increase skeletal muscle blood flow, glycogenolysis, and lipolysis. (Choices C and D) Calmodulin and myosin light-chain kinase are elements of the contractile mechanism in smooth muscle, not skeletal muscle. Educational objective: The contractile mechanism in skeletal muscle depends on proteins (myosin II, actin, tropomyosin, and troponin) as well as calcium ions.

A 17-year-old boy is referred to a clinic for ongoing muscle weakness. He has noticed difficulty climbing stairs at home over the last several months, and he recently stopped playing basketball due to difficulty lifting the ball over his head. The patient takes no medications and does not know of any family members with muscle weakness. On neurologic examination, he is unable to lift his arms or legs against resistance when fully outstretched. He undergoes a muscle biopsy to evaluate the cause of his proximal muscle weakness. Electron microscopy of the specimen reveals sparse transverse tubules in some of the muscle fibers. Which of the following is the most likely consequence of this patient's biopsy findings? Decremental force generation on repeated muscle stimulation Impaired ATP production during active muscle contraction Impaired relaxation after sustained muscle contraction No contraction of the myocyte in response to increased intracellular Ca2+ Uncoordinated contraction of myofibrils within affected muscle fibers

ID=824 Transverse tubules (T-tubules) are invaginations of the muscle cell membrane (sarcolemma) located in close proximity to the terminal cisterns of the sarcoplasmic reticulum. They are found in striated (eg, cardiac, skeletal) muscle and contain voltage-gated L-type calcium channels (dihydropyridine receptors) adjacent to the ryanodine receptors on the terminal cisterns. During muscle contraction, T-tubules allow the depolarization impulse to rapidly propagate through the interior of the muscle fiber. This ensures that calcium release from the sarcoplasmic reticulum occurs uniformly throughout the fiber, allowing for synchronized contraction of myofibrils in each muscle cell. This patient most likely has a form of limb girdle muscular dystrophy due to a mutated sarcolemma protein (eg, caveolin) affecting excitation-contraction coupling. Decreased numbers of functional T-tubules in affected muscle fibers lead to uncoordinated contraction of myofibrils, which manifests as muscle weakness. (Choice A) Decremental force generated on repeated stimulation characterizes myasthenia gravis, an autoantibody disorder inhibiting postsynaptic acetylcholine receptors in the neuromuscular junction. A constant low response to repeated nerve stimulation is more likely if T-tubules are lacking. (Choice B) ATP fuels myosin movement along actin filaments (causing contraction) and drives the pumps that transport calcium back into the sarcoplasmic reticulum (causing relaxation). McArdle disease is one cause of impaired ATP production and occurs due to a failure of muscle glycogen breakdown. (Choice C) Impaired relaxation after sustained contraction occurs in myotonic dystrophy due to a trinucleotide repeat expansion altering myotonin-protein kinase. This protein facilitates myosin head detachment from the actin filament to enable muscle relaxation; T-tubules do not play a role. (Choice D) A mutation in troponin C may block its response to intracellular calcium and prevent muscle contraction. However, muscle fibers with decreased numbers of T-tubules will maintain limited contractility. Educational objective: Transverse tubules (T-tubules) are invaginations of the sarcolemma that transmit depolarization signals to the sarcoplasmic reticulum to trigger the release of calcium and induce muscle contraction. The uniform distribution of T-tubules in striated muscle fibers ensures that each myofibril contracts at the same time, which is necessary for efficient contraction.

A 42-year-old weightlifter is preparing for an upcoming strength competition. After lifting a series of progressively heavier weights, he decides to increase the weight on the exercise bar to 175 kg (385 lbs), the heaviest lift he has ever attempted. He is able to hold the weight over his head for several seconds. However, his arms suddenly and involuntarily give way and he drops the weight to the ground. Which of the following structures was most likely responsible for the sudden muscle relaxation? A-delta nerve endings Golgi tendon organ Intrafusal muscle fibers Pacinian corpuscles Ruffini's end organs

ID=8266 The Golgi tendon organs (GTOs) are sensory receptors located at the junction of the muscle and tendon that are innervated by group Ib sensory axons. GTOs are connected in series with the contracting extrafusal skeletal muscle fibers. When a muscle actively contracts against resistance, the increase in tension is transmitted through the tendon activating the GTOs in the process. In contrast, GTOs are relatively insensitive to changes in muscle length because the lengthening that occurs when a muscle is passively stretched takes place primarily in the muscle fibers and not in the tendon. The Ib sensory axons from the GTOs contact inhibitory interneurons in the spinal cord, which in turn synapse with the α motor neurons that innervate the same muscle. Thus, the Golgi tendon circuit is a negative feedback system that regulates and maintains muscle tension. When a muscle exerts too much force, the GTOs inhibit contraction of the muscle, causing sudden muscle relaxation. This prevents damage to the musculoskeletal system. (Choice A) A-delta fibers are thin, myelinated nerve fibers whose free nerve endings detect temperature and nociceptive stimuli. They are associated with acute (sharp) pain and constitute the afferent portion of the reflex arc that mediates withdrawal from noxious stimuli (e.g., retracting the hand away from a hot stove). (Choice C) In contrast to the in-series arrangement of GTOs, muscle spindles (intrafusal muscle fibers) are connected in parallel with extrafusal fibers. They are innervated by group Ia and group II sensory axons and are sensitive to changes in muscle length. Muscle spindles mediate the stretch reflex (myotatic reflex), which is commonly tested (deep tendon reflexes) during the neurologic examination. When a muscle is stretched, there is monosynaptic reflex activation of the α motor neuron (of the same muscle), causing contraction that resists the stretch. (Choices D and E) Pacinian corpuscles are rapidly adapting mechanoreceptors located in the subcutaneous tissue of the skin as well as the mesentery, peritoneum, and joint capsules. Ruffini's end organs are slowly adapting mechanoreceptors that exist in the skin, subcutaneous tissue, and joint capsule. Both help to mediate touch, proprioception, and vibratory sensation and are innervated by myelinated A-beta fibers. Educational objective: The muscle spindle system is a feedback system that monitors and maintains muscle length, while the Golgi tendon system is a feedback system that monitors and maintains muscle force. GTOs are exquisitely sensitive to increases in muscle tension but are relatively insensitive to passive stretch.

A 28-year-old man is undergoing a laparoscopic appendectomy for acute appendicitis. Shortly after induction of anesthesia, the surgeon reports difficulty maintaining abdominal insufflation due to severe muscle rigidity. Pulse is 130/min and the ECG tracing shows sinus tachycardia. Physical examination reveals diffuse muscle stiffness. Inhibition of which of the following is most likely to improve this patient's condition? Acetylcholine degradation in cholinergic synapses Alpha-receptor-mediated sympathetic activity GABA-mediated neuron hyperpolarization Ligand-receptor interaction in cholinergic synapses Sarcoplasmic calcium release in skeletal muscles

ID=885 During laparoscopic procedures, small incisions are created to insert instruments into the abdominal cavity. Gas is insufflated to allow better visualization and easier access to organs while operating. Difficulty maintaining abdominal insufflation due to muscle contraction is often caused by insufficient doses of paralytic agents. However, if the patient has been appropriately medicated, sustained muscle contraction, especially when widespread, may be due to malignant hyperthermia (MH). MH is often due to abnormal ryanodine receptors (RYR1) in the skeletal muscle. On exposure to certain anesthetic agents (typically succinylcholine and inhalational gases), these abnormal receptors allow the unregulated passage of calcium from the sarcoplasmic reticulum into the intracellular space. As a result of excessive calcium accumulation, patients develop sudden-onset, widespread muscle rigidity. Sustained muscle contraction produces excessive CO2, leading to hypercarbia and acidosis; depletes cellular oxygen, leading to lactic acid production; and causes muscle breakdown, leading to rhabdomyolysis. Hyperthermia is often a later clinical sign that develops as the increased cellular metabolism overwhelms the body's ability to dissipate heat. Many pharmacologic muscle relaxants work upstream from the pathologic process of excessive calcium release and are therefore ineffective during an acute MH crisis. Dantrolene is used to treat MH because it directly inhibits intracellular calcium release from the abnormal ryanodine receptor (RYR1). (Choice A) Neostigmine is an acetylcholinesterase inhibitor that prevents the degradation of acetylcholine in the neuromuscular junction. Therefore, it could increase, rather than decrease, the strength of muscle contractions. (Choice B) Alpha-adrenergic antagonists (eg, phentolamine, phenoxybenzamine) inhibit alpha-receptor-mediated sympathetic activity. They are used primarily to lower blood pressure by decreasing vascular tone; they have minimal effect on skeletal muscle. (Choices C and D) Baclofen is a GABA-B agonist that increases neuron hyperpolarization in the CNS to relax muscles. Nondepolarizing neuromuscular blocking agents (eg, rocuronium) are nicotinic receptor antagonists that inhibit the action of acetylcholine at the motor endplate. Both agents work upstream from intracellular calcium release and therefore have minimal impact on muscle contraction during an MH crisis. Educational objective: Malignant hyperthermia presents with widespread muscle rigidity after administration of inhalation anesthetics and/or succinylcholine to genetically susceptible individuals. It is treated with dantrolene, which blocks ryanodine receptors to prevent the release of calcium from the sarcoplasmic reticulum into the cytoplasm of skeletal muscle fibers.

A 25-year-old woman comes to the hospital due to worsening dyspnea. The patient was previously healthy; 2 months ago, she delivered a healthy baby boy at 38 weeks gestation. Her arterial blood gas analysis shows evidence of hypoventilation. During further evaluation, she is asked to take several deep breaths, and the following physiologic information is obtained: ID=1657 Which of the following locations is the most likely site of this patient's disease? Brain stem Nasopharynx Neuromuscular junction Pericardial space Peripheral nerves

Intrapleural pressure (ie, between the lung and chest wall pleura) is slightly negative at rest due to the inward elastic recoil of the lungs and outward recoil of the chest wall. During inspiration, diaphragmatic contraction is primarily responsible for creating a more negative intrapleural pressure that causes expansion of the thoracic cavity, drawing air into the lungs. Passive expiration occurs with respiratory muscle relaxation and relies on the natural inward elastic recoil of the lungs to expel air as the intrapleural pressure returns to baseline. The graph shows progressive attenuation of the intrapleural pressure change that occurs during deep, repetitive inspiration. Because there is no apparent decrement in phrenic nerve activity, this patient's progressively weaker diaphragmatic contractions are likely due to excessive skeletal muscle fatigue (eg, severe obstructive or restrictive lung disease) or impaired neuromuscular transmission. In myasthenia gravis, autoantibodies against postsynaptic acetylcholine receptors cause fatigable skeletal muscle weakness as acetylcholine stores within the presynaptic nerve terminal become progressively depleted with repetitive stimulation. Women in the postpartum period (like this patient) are at particularly high risk of developing myasthenia gravis. (Choice A) Impairment of the respiratory control centers in the brain stem can result in decreased frequency and/or amplitude of involuntary respirations (Ondine curse). Voluntary breathing would be unaffected. (Choice B) Nasopharyngeal lesions are unlikely to cause inspiratory airflow obstruction severe enough to cause rapid diaphragmatic fatigue as the oropharynx remains unobstructed (allowing the patient to breath from the mouth). (Choices D and E) Metastatic disease to the pericardium could compress the phrenic nerve due to its location along the pericardium. In addition, the phrenic nerve can be affected by a peripheral neuropathy. However, both diseases cause uniform weak inspirations during maximal voluntary ventilation rather than progressively weaker diaphragmatic contractions. Educational objective: Progressively weakening diaphragmatic contractions during maximal voluntary ventilation with intact phrenic nerve stimulation indicate neuromuscular junction pathology (eg, myasthenia gravis) and/or abnormally rapid diaphragmatic muscle fatigue (eg, restrictive lung or chest wall disease).

A 34-year-old man comes to the clinic due to frequent clumsiness. For the last 6 months, he has had repeated minor injuries from walking into doorways and stationary objects. The patient was also reprimanded at work for causing his supervisor to spill coffee when he bumped into her in the hallway. Medical history is notable for recurrent renal stones; following a subsequent diagnostic evaluation, the patient underwent definitive treatment with neck surgery 3 years ago. Family history is unavailable as the patient was adopted. Physical examination reveals bitemporal visual field defects. Cranial nerves, motor strength, and deep-tendon reflexes are otherwise normal. In addition to brain imaging, this patient should also be screened for which of the following tumors? Adrenal Bone Breast Colonic Pancreatic

Multiple endocrine neoplasia type 1 (MEN1) is caused by mutations in the MEN1 tumor suppressor gene and is characterized by tumors of the parathyroid gland, pituitary, and pancreas (the "3 Ps"). Primary hyperparathyroidism is often the initial manifestation and usually presents with asymptomatic hypercalcemia or renal stones. Prolactin-secreting adenomas (prolactinomas) are the most common pituitary tumors. In premenopausal women, prolactinomas typically present with metabolic symptoms (eg, menstrual irregularities, galactorrhea); in men and postmenopausal women they usually present later when tumor enlargement leads to mass effect symptoms such as headache or bitemporal visual field defects (causing clumsiness in this patient) due to compression of the optic chiasm. Entero-pancreatic neuroendocrine tumors are the third major tumor type in MEN1. Gastrinoma (leading to Zollinger-Ellison syndrome) is the most common, followed by insulinoma and nonfunctional tumors. These tumors are often metastatic and are a frequent cause of tumor-related death in MEN1. This patient has visual field defects consistent with a pituitary mass lesion, which warrants cerebral imaging. In addition, he has a history of symptomatic hyperparathyroidism. Given these features of MEN1, he is also at increased risk for entero-pancreatic neuroendocrine tumors. (Choice A) Pheochromocytoma is an adrenal medullary tumor that occurs commonly in MEN2A and 2B. However, this patient's visual field defect suggests a pituitary tumor, which is more characteristic of MEN1 (not associated with pheochromocytoma). (Choice B) Primary bone tumors are not increased in MEN1, although malignant neuroendocrine tumors may occasionally metastasize to bone. (Choice C) The major inherited syndromes associated with breast cancer include BRCA1 and BRCA2 mutations (breast, ovarian, uterine/fallopian tube, pancreatic cancers). Less common syndromes include Cowden syndrome, Li-Fraumeni syndrome, and Peutz-Jeghers syndrome. Hyperparathyroidism and pituitary tumors are not prominent manifestations of any of these syndromes and are more consistent with MEN1. (Choice D) Familial adenomatous polyposis is associated with an increased risk of colon cancer and with adrenal adenomas and thyroid cancer. Educational objective: Multiple endocrine neoplasia type 1 syndrome is characterized by tumors of the pituitary, parathyroid gland, and pancreas (the "3 Ps").

A 3-year-old boy is brought to the office by his parents. They say that the child cannot keep up with his cousin, who is the same age. The boy climbs up from a squat by using his hands to push off the ground and his lower extremities. His calves appear enlarged on physical examination. This patient's condition is most likely associated with which of the following? Endomysial inflammatory infiltration Endoneural inflammatory infiltration Mutation affecting a muscle ion channel protein Mutation affecting a myelin protein Mutation affecting a sarcolemma-cytoskeleton linker protein Mutation affecting a sarcomere protein

Muscular dystrophy is a term that applies to the various diseases that manifest with progressive muscular weakness. Deletions of the dystrophin gene that encodes the dystrophin protein on X chromosome p21 are the most common mutation in Duchenne muscular dystrophy (DMD) and Becker muscular dystrophy (BMD). Deletions that are not a multiple of 3 change the reading frame, causing a frameshift mutation, which results in a nonfunctional protein and severe clinical manifestations (DMD). In contrast, deletions that are a multiple of 3 base pairs preserve the reading frame, resulting in a truncated but functional protein and a milder clinical phenotype (BMD). Dystrophin is a structural component of skeletal muscle fibers that provides mechanical stability to the sarcolemma. It links a component of the cytoskeleton (actin) to transmembrane proteins (α- and β-dystrophiglycans) that are connected to the extracellular matrix. Loss of dystrophin results in cellular injury (myonecrosis). On light microscopy, there is segmental degeneration and regeneration of the myofibers with marked variation in size (both atrophic and hypertrophic fibers). Over time, the muscle tissue undergoes progressive fatty replacement. On clinical examination, the presence of Gowers sign and calf enlargement in a boy age 2-5 is classic for DMD. Calf hypertrophy allows affected children to overcome proximal muscle weakness, but it is later replaced by fat and connective tissue (pseudohypertrophy). The Gowers sign involves the use of one's hands to rise from a squat or from a chair to compensate for proximal muscle weakness. (Choice A) Endomysial inflammatory infiltration is found on muscle biopsy in polymyositis. Dermatomyositis is associated with perifascicular inflammation. Both diseases cause proximal muscle weakness but not distal muscle hypertrophy. (Choice B) Endoneural inflammatory infiltration is characteristic of Guillain-Barré syndrome, which classically manifests as ascending flaccid paralysis and hyporeflexia. (Choice C) Examples of ion channel myopathies include hyper- and hypokalemic periodic paralysis, which present with episodic, painless muscle weakness. (Choice D) Charcot-Marie-Tooth disease is due to mutation of the genes responsible for myelin synthesis. Affected patients frequently present with distal muscle weakness, sensory loss, and atrophy of the calf muscles (producing the characteristic stork leg deformity). (Choice F) Mutations involving sarcomere proteins are responsible for hypertrophic cardiomyopathy and certain forms of familial dilated cardiomyopathy. Educational objective: Duchenne muscular dystrophy is an X-linked recessive myopathy that manifests with proximal muscle weakness and enlargement of the calf muscles in boys age 2-5. It most often results from frameshift deletions affecting the dystrophin gene. Dystrophin provides a stabilizing interaction between the sarcolemma and the intracellular contraction apparatus, and disruption of the protein results in membrane damage and myonecrosis.

A 31-year-old woman is brought to the hospital due to respiratory difficulty. She has had trouble performing her daily activities for the past few weeks due to fatigue. In particular, her arms seem to "just drop" when she combs her hair. The patient's symptoms are generally better in the morning and worsen throughout the day. She does not use tobacco, alcohol, or illicit drugs. Examination shows diplopia. Chest auscultation shows normal heart and breath sounds. Deep tendon reflexes are normal. Babinski sign is absent. The results of arterial blood gas analysis are as follows: pH 7.32 pCO 260 mm Hg pO 260 mm Hg Which of the following is most likely abnormal in this patient? Acetylcholine release from presynaptic neurons Acetylcholinesterase activity Calcium entry into presynaptic neurons Number of acetylcholine receptors Reuptake of acetylcholine

Myasthenia gravis (MG) is an acquired autoimmune disorder characterized by fatigable, fluctuating weakness of the skeletal muscles. The most commonly affected muscles include ocular (diplopia, ptosis), bulbar (dysarthria, fatigable chewing), facial (loss of smile), and proximal (neck/shoulder weakness) muscles. However as the disease progresses, respiratory failure (↓ pH, ↑ pCO2, ↓ O2) can occur due to diaphragmatic weakness and patients may require intubation. MG is caused by an autoantibody-mediated, T cell-dependent attack on the acetylcholine receptors (AChRs) of the postsynaptic neuromuscular junction. Antibody-mediated blockade of the active site of AChRs prevents acetylcholine from binding and triggers endocytosis of AChRs (receptor internalization) and complement-mediated postsynaptic membrane damage. Over time, these changes result in reduced numbers of AChRs and impaired neuromuscular transmission. (Choice A) Botulinum toxin irreversibly blocks acetylcholine release from presynaptic membranes of neuromuscular junctions. After exposure to the toxin, patients may develop cranial nerve abnormalities (including ptosis and facial weakness), symmetric descending weakness, and respiratory failure. However, autonomic abnormalities are common, and symptoms are rapidly progressive (hours rather than weeks) and do not worsen with repetition. (Choices B and E) The action of acetylcholine at the postsynaptic membrane is terminated by acetylcholinesterase, not by acetylcholine reuptake. Organophosphates (eg, pesticides, sarin gas) irreversibly inactivate acetylcholinesterase, causing cholinergic toxicity (eg, increased glandular secretions, widespread smooth muscle activation). Muscle weakness, including respiratory failure, can occur due to depolarizing blockade at the neuromuscular junction; however, other symptoms (eg, diarrhea, urination, miosis, bradycardia, emesis, salivation) would be expected. (Choice C) Lambert-Eaton myasthenic syndrome is due to autoantibodies directed against the voltage-gated calcium channels on presynaptic nerve terminals, leading to reduced calcium entry and impaired acetylcholine release. This syndrome commonly occurs in association with malignancy, particularly small cell lung cancer (which would be atypical in this young female), and generally causes proximal extremity weakness that is worse in the mornings and improves with repetition. Autonomic findings (eg, dry mouth, erectile dysfunction in males) are frequently seen and can be an important clue to diagnosis. Educational objective: Myasthenia gravis is an autoimmune disorder caused by an autoantibody-mediated attack on the acetylcholine receptors of the postsynaptic neuromuscular junction, leading to a reduced number of acetylcholine receptors. It is characterized by fatigable weakness of the skeletal muscles; respiratory failure can occur due to diaphragmatic weakness.

A 66-year-old man comes to the office with diplopia that progressively worsens toward the end of the day. The patient has no shortness of breath, trouble swallowing, or generalized weakness. He is currently receiving low-dose pyridostigmine for treatment of his condition and claims good compliance without significant side effects. Physical examination reveals fatigable asymmetric ptosis and binocular horizontal diplopia. Infusion of edrophonium elicits a significant improvement in his symptoms. Which of the following is the best next step in management of this patient? Add atropine Administer pralidoxime Change long-term therapy to edrophonium Increase pyridostigmine dosage Stop pyridostigmine temporarily

Myasthenia gravis is an autoimmune disease caused by autoantibodies directed against nicotinic acetylcholine receptors on the postsynaptic membrane of the neuromuscular junction. Action potential propagation is impaired in these patients as antibody binding leads to receptor degradation and complement-mediated damage of the postsynaptic membrane. Patients typically present with fluctuating weakness that worsens over the course of the day and most often involves the extraocular (eg, ptosis, diplopia) and bulbar (eg, fatigable chewing, dysphagia) muscles. Acetylcholinesterase (AChE) is an enzyme that degrades acetylcholine in the synaptic cleft. Long-acting AChE inhibitors (eg, pyridostigmine) are used in the symptomatic treatment of myasthenia gravis as they increase acetylcholine concentration in the neuromuscular junction, prolonging its effect on the postsynaptic membrane. Exacerbations of myasthenia gravis in patients being treated with AChE inhibitors may occur for 2 reasons. First, if the patient is undertreated (receives suboptimal doses of medication), not enough acetylcholine is available in the neuromuscular junction (myasthenic crisis). Infusion of the short-acting AChE inhibitor edrophonium (Tensilon test) increases neuromuscular transmission and can provide temporary improvement in symptoms (Choice C). The pyridostigmine dose should be increased in the long-term management of these patients. The second (rare) cause of exacerbation is cholinergic crisis. In this condition, a patient receives inappropriately high doses of AChE inhibitor and has excessive acetylcholine in the synaptic cleft. This causes excessive stimulation of the skeletal muscles and results in muscle that is refractory to future impulses. Cholinergic crisis also presents with muscle weakness; however, because the neuromuscular junction becomes insensitive to acetylcholine, edrophonium infusion produces no improvement in symptoms. AChE inhibitors should be temporarily discontinued in patients with cholinergic crisis (Choice E). (Choice A) Atropine is a muscarinic acetylcholine receptor antagonist that may be used in organophosphate poisoning and as a mydriatic agent (dilates pupils). However, it does not affect nicotinic receptors and is not used to treat myasthenic crisis. (Choice B) Pralidoxime is also used in organophosphate poisoning. Pralidoxime reverses the effects of both muscarinic and nicotinic overstimulation by "regenerating" AChE in the neuromuscular junction. Educational objective: Exacerbation of myasthenia gravis in a patient treated with long-acting acetylcholinesterase inhibitors (eg, pyridostigmine) occurs due to myasthenic or cholinergic crisis. The edrophonium (Tensilon) test helps to differentiate these 2 conditions. Clinical improvement after edrophonium administration indicates that the patient is undertreated (myasthenic crisis).

A 55-year-old man comes to the office for evaluation of chronic muscle weakness. Over the past several months, he has had increasing difficulty walking up stairs and lately has had trouble removing objects out of the overhead cabinets in his kitchen. Temperature is 36.7 C (98 F), blood pressure is 125/80 mm Hg, and pulse is 78/min. On examination, the patient has symmetric proximal muscle weakness and mild muscle tenderness. There is no rash. Muscle biopsy reveals an endomysial mononuclear infiltrate and patchy muscle fiber necrosis. An autoantibody directed against which of the following antigens is most likely to be seen in this patient? Acetylcholine receptor Cardiolipin Desmoglein Mitochondria Presynaptic calcium channel tRNA synthetase Smooth muscle

Polymyositis is an inflammatory myopathy that typically presents in middle age with an insidious onset of symmetric proximal muscle weakness affecting the upper and lower extremities without significant pain. Dermatomyositis is a similar disorder associated with characteristic skin findings. Both polymyositis and dermatomyositis may occur independently or as a paraneoplastic manifestation of an underlying malignancy (especially adenocarcinoma). Muscle enzyme (eg, creatine kinase, aldolase) levels in inflammatory myopathies are invariably elevated. Autoantibodies, especially antinuclear antibodies, are also present in most cases; anti-histidyl-tRNA synthetase (anti-Jo-1) antibodies are less sensitive but more specific for dermatomyositis and polymyositis. A biopsy can differentiate polymyositis from dermatomyositis, which typically shows an endomysial inflammation without prominent vascular involvement in a scattered or patchy distribution (in contrast, dermatomyositis causes perifascicular inflammation in a segmental pattern without vasculopathy). (Choices A and E) Acetylcholine receptor-binding antibodies occur in myasthenia gravis, which is characterized by episodic weakness that initially affects the ocular/bulbar musculature and worsens with repetition. Antibodies to voltage-gated calcium channels cause Lambert-Eaton myasthenic syndrome (proximal muscle weakness that improves with isometric contraction), which is often associated with small cell lung cancer. Muscle biopsy in these conditions is normal. (Choice B) Anticardiolipin antibodies are typically seen in antiphospholipid antibody syndrome, which is characterized by recurrent vascular thrombosis and/or pregnancy loss. This syndrome is associated with systemic lupus erythematosus, although it may occur independently. (Choice C) Autoantibodies to desmoglein 1 and 3 are seen in patients with pemphigus vulgaris, which is characterized by skin blistering and desquamation. (Choices D and G) Antimitochondrial antibodies are seen in patients with primary biliary cholangitis, which presents with cholestatic symptoms (eg, pruritus, jaundice, steatorrhea) and abnormal hepatic enzyme levels. Anti-smooth muscle antibodies are seen in autoimmune hepatitis, which most commonly presents as chronic progressive hepatitis in middle-aged women. Educational objective: Polymyositis and dermatomyositis are characterized by symmetric proximal muscle weakness and are associated with antinuclear and anti-tRNA synthetase (anti-Jo-1) autoantibodies. Biopsy in polymyositis shows patchy endomysial inflammatory infiltrate (ie, direct invasion of individual muscle fibers), whereas dermatomyositis causes perifascicular inflammation (ie, localized around blood vessels and the septa between muscle fascicles).

An 8-year-old boy is evaluated in the office due to difficulty walking. The patient has problems running, jumping, and walking up steps. There is no history of neurological disorders in the family. Physical examination reveals weakness of lower extremity muscles bilaterally. Electron microscopy of a biopsy sample taken from calf muscle is shown below. ID=1664 Which of the following labeled regions contains only thick filaments and no thin filaments? A B C D E

The H band is the region of the sarcomere containing only thick (myosin) filaments. On electron microscopy, the H band is a portion of the A band that straddles the M line. The A band corresponds to the thick filaments in the sarcomere and includes portions overlapped by thin (actin) filaments (Choice B). During muscle contraction, the thin filaments slide over the thick filaments toward the M line, reducing the length of the H band. The A band always remains the same length. (Choice A) The Z line is typically distinctly darker than the remainder of the sarcomere. Thin filaments, which are composed of actin, tropomyosin, and troponin, anchor at the Z line. (Choice C) The segment of the sarcomere between the H and I bands is where thick and thin filaments overlap. (Choice D) The I band contains the Z line and only those sections of the thin filaments that do not overlap with thick filaments. During muscle contraction, both the I band and H band decrease in length. Educational objective: The H band is the region of the sarcomere that contains only thick (myosin) filaments. The H band is the part of the A band (which is on either side of the M line) where thick filaments have no overlapping thin (actin) filaments.

A 26-year-old woman is being evaluated for a possible inherited disorder. She has a 6-year history of generalized tonic-clonic seizures, and a year ago, she had partial loss of vision due to an occipital infarction. Her mother has chronic intermittent muscle weakness and lactic acidosis, and her maternal uncle has hemiplegia. Skeletal muscle biopsy of the patient shows ragged-appearing muscle fibers. After further evaluation, all the affected family members are found to suffer from the same inherited disease. Pedigree analysis is shown in the image below. ID=1935 Which of the following is the most likely explanation for the variability of clinical manifestations in the affected family members? Anticipation Heteroplasmy Incomplete penetrance Mosaicism Uniparental disomy

The neuromuscular lesions, ragged skeletal muscle fibers, and lactic acidosis in these family members suggest mitochondrial encephalomyopathy. Mitochondrial disorders follow a maternal inheritance pattern because an embryo's mitochondria are inherited from the ovum only. Mitochondria are responsible for ATP production via oxidative phosphorylation, which is why mitochondrial defects tend to cause lactic acidosis and primarily affect tissues with the highest metabolic rates (eg, neural tissue, muscular tissue). Although many mitochondrial proteins are coded for in the nuclear genome, mitochondria also contain their own genome, which is vulnerable to mutations. Each cell has hundreds of mitochondria, and defects in their genome may occur in any number of the mitochondria within the cell. Heteroplasmy describes the condition of having different mitochondrial genomes within a single cell. The severity of mitochondrial diseases is often directly related to the proportion of abnormal to normal mitochondria within a patient's cells. (Choice A) Anticipation refers to increased severity or earlier onset of an inherited disease in successive generations. It is commonly seen in autosomal and X-linked trinucleotide repeat disorders due to trinucleotide amplification during gametogenesis. (Choice C) Penetrance is the probability that a person with a given genotype will express its associated phenotype. If all individuals with a given gene express its phenotype, that gene is said to have complete penetrance. (Choice D) Mosaicism refers to the presence of 2 or more cell lines, each with a unique nuclear genome, within the same individual. While patients with combined somatic and germline mosaicism can demonstrate disease traits and also pass the disease on to their offspring, mosaicism would not explain the pattern of female-only transmission. (Choice E) Uniparental disomy occurs when both members of a chromosomal pair are inherited from one parent, which can cause problems due to genomic imprinting. For instance, although most often due to chromosomal deletions, uniparental disomy can also cause Prader-Willi and Angelman syndromes due to loss of expression of maternal/paternal imprinted components of a critical region of chromosome 15. Educational objective: The presence of lactic acidosis and ragged skeletal muscle fibers histologically suggests a mitochondrial myopathy. Variable clinical expressions in affected family members can occur due to heteroplasmy, which is the coexistence of distinct versions of mitochondrial genomes in an individual cell.

A 46-year-old woman comes to the office due to dysphagia. The patient has had difficulty swallowing both liquids and solid foods as well as frequent episodes of regurgitation and cough while eating. She has also had weakness in her extremities and a rash. The patient has no prior medical problems and takes no medications. Vital signs are within normal limits. Physical examination shows an erythematous eruption on the upper eyelids. The oropharynx is clear and cardiopulmonary and abdominal examinations are unremarkable. She has difficulty lifting her arms above her head and standing from a sitting position. Which of the following is the most likely cause of this patient's dysphagia? Atrophy and fibrous replacement of distal esophageal muscularis propria Diffuse eosinophil-predominant inflammation of the esophageal mucosa Inflammation and degeneration of the esophageal myenteric plexus Outpouching of hypopharyngeal mucosa through weakened muscle Perifascicular atrophy of muscle fibers in the proximal esophageal muscularis propria

This patient has painless proximal muscle weakness; in association with the heliotrope rash on her eyelids, this presentation is consistent with dermatomyositis, an inflammatory myopathy that primarily affects striated muscle and skin. Diagnostic findings include elevated muscle enzymes (eg, creatinine kinase) and positive autoantibodies (ie, antinuclear antibody [high sensitivity, low specificity], anti-Jo-1 [low sensitivity, high specificity]). Muscle biopsy in dermatomyositis shows perimysial inflammatory infiltrates and atrophy involving the fibers around the periphery of muscle fascicles (perifascicular atrophy). The oropharynx and upper esophagus also contain skeletal muscle that can be affected in dermatomyositis and polymyositis, leading to dysphagia and pulmonary aspiration. Other important complications of inflammatory myopathies include interstitial lung disease and myocarditis (due to involvement of striated cardiac muscle). (Choice A) Progressive systemic sclerosis (scleroderma) commonly affects the esophagus, leading to atrophy and fibrous replacement of the distal esophageal muscularis propria. Clinical features include dysphagia, reflux esophagitis, and aspiration. However, the dermal manifestations include sclerodactyly, digital ulcers, and calcinosis cutis, rather than a heliotrope rash. Also, dermatomyositis affects only the striated muscle in the oropharynx and upper esophagus (eg, choking, aspiration), whereas scleroderma can affect the lower esophagus. (Choice B) Eosinophilic esophagitis typically presents with dysphagia, epigastric pain, recurrent esophageal reflux, and food impaction. Skin rash and extremity weakness would be unexpected. (Choice C) Achalasia is caused by inflammation and degeneration of the esophageal myenteric plexus, resulting in impaired relaxation of the smooth muscle in the distal esophagus and lower esophageal sphincter. It typically presents with dysphagia, regurgitation, and heartburn but is not associated with rash or skeletal muscle weakness. (Choice D) Zenker diverticulum develops immediately above the upper esophageal sphincter, with posterior mucosal herniation between the fibers of the cricopharyngeus muscle. Although it presents with dysphagia, regurgitation, and aspiration, it is not associated with rash or muscular weakness. Educational objective: Dermatomyositis is characterized by proximal muscle weakness resembling polymyositis, with additional inflammatory features affecting the skin (eg, heliotrope rash, Gottron papules). In both conditions, involvement of striated muscle in the oropharynx and heart can lead to dysphagia, pulmonary aspiration, and myocarditis.

Electric stimulation of the musculocutaneous nerve causes a rapid twitch of the biceps muscle. Electron microscopy shows numerous vesicles within the musculocutaneous nerve terminals. In order for these vesicles to release their contents into the synaptic cleft, which of the following substances is required? Sodium Calcium cAMP IP3 DAG cGMP

The synaptic vesicles in motor neuron presynaptic terminals contain acetylcholine (ACh). When an action potential depolarizes the presynaptic terminal membrane, voltage-gated calcium channels open, causing an influx of extracellular calcium into the synaptic terminal. This calcium current stimulates fusion of the presynaptic vesicles with the plasma membrane and exocytosis of ACh. The nicotinic ACh receptors on the postsynaptic skeletal muscle end plate are ligand-gated sodium and potassium ion channels. The increased sodium and potassium conductance that occurs with ACh binding generates a motor end plate potential that signals for depolarization of the muscle cell membrane. ACh in the synaptic cleft is degraded by acetylcholinesterase. (Choice A) Rapid sodium influx is responsible for the upstroke of the action potential in skeletal muscle, smooth muscle, cardiac muscle and Purkinje fibers. In contrast, the initial upstroke of the action potential in the SA node depends on calcium influx. (Choice C - F) Second messengers do not play a role in the release of synaptic vesicle contents (ACh) at the neuromuscular junction. Educational Objective: Acetylcholine release from presynaptic terminal vesicles at the neuromuscular junction depends upon the influx of extracellular calcium into the presynaptic terminal. Calcium influx into the nerve terminal occurs following neuronal depolarization and opening of voltage-gated calcium channels.

A 50-year-old man comes to the office for evaluation of a mass on his back. He has had the mass for several years, and it has increased only slightly in size during that time. The patient has 2 other similar lesions, one on his arm and the other on his leg. The lesions are painless, and he has no other symptoms. Vitals signs are normal. Examination shows multiple subcutaneous, soft, freely mobile, and nontender masses. The lesion on his back is shown in the image. ID=15665 Histologic examination of the mass is most likely to show which of the following findings? Mature adipocytes arranged in lobules and separated by fibrous septa Pleomorphic malignant cells producing new woven bone Septal panniculitis with a mixed cellular infiltrate of lymphocytes and histiocytes Spindle cells with wavy nuclei embedded in collagen stroma

This patient has a soft, mobile, subcutaneous mass consistent with a lipoma. Lipomas are very common, benign tumors that arise from the subcutaneous fat. They typically occur in middle-aged adults and are generally stable or enlarge slowly over time. The overlying epidermis is normal. The diagnosis of lipoma is usually easy to make on clinical examination. However, if necessary, excisional biopsy can be performed and shows well-differentiated, mature adipocytes with an intact fibrous capsule. Because lipomas are generally harmless, most can be left in place, although excision can be considered for cosmetic reasons or patient discomfort. (Choice B) Osteosarcoma is usually seen in children and young adults; it is uncommon in older adults, in which it is often associated with Paget disease of bone. It presents with local pain and swelling; the mass, if palpable, is nonmobile. Histopathology shows pleomorphic malignant cells with hyperchromatic nuclei forming neoplastic osteoid or bone. (Choice C) Erythema nodosum presents acutely as tender, subcutaneous nodules in the lower extremities. It occurs in association with a variety of infectious (eg, herpes simplex) and inflammatory (eg, sarcoidosis) conditions. Histopathology shows inflammation in the subcutaneous fat (panniculitis) with edema, fibrinous exudate, and a variable cellular infiltrate (neutrophils, lymphocytes, histiocytes). (Choice D) Cutaneous squamous cell carcinoma typically forms an enlarging epidermal nodule in sun-exposed areas. The lesions often develop a thickened, rough surface or ulcerate with crusting and bleeding. Pathologic findings include sheets of eosinophilic squamous cells with keratin pearls and intercellular bridges. (Choice E) Cutaneous neurofibromas usually manifest during early adolescence as raised, fleshy tumors (<2 cm) that often increase in size and number with age. They are composed primarily of Schwann cells (spindle-shaped cells with wavy or serpentine nuclei), fibroblasts, and perineural cells. Educational objective: Lipomas are common, benign tumors that arise from the subcutaneous fat and present as soft, mobile masses that are stable or enlarge slowly over time. The diagnosis is usually made clinically, but histopathology shows well-differentiated, mature adipocytes with a fibrous capsule.

A 42-year-old woman is hospitalized for surgical removal of a mediastinal mass. The patient has had intermittent episodes of double vision, difficulty chewing food, and weakness of the extremities for the past several months. During evaluation she was found to have a circumscribed anterior mediastinal mass. Preoperative nerve conduction studies revealed a decremental response to repetitive motor nerve stimulation. She has no other medical conditions and does not use tobacco, alcohol, or illicit drugs. The patient has no drug allergies. Compared to a healthy individual, which of the following anesthesia-related medications is most likely to have increased potency in this patient? Isoflurane Midazolam Neostigmine Atropine Vecuronium

This patient has an anterior mediastinal mass and a nerve conduction study consistent with myasthenia gravis (MG), an autoimmune disease caused by autoantibodies against postsynaptic nicotinic acetylcholine receptors of the neuromuscular junction. Antibody-induced degradation leads to fewer functional receptors, which may be insufficient to generate muscle action potentials despite normal acetylcholine release. Weakness (most commonly of the extraocular, bulbar, and proximal muscles) worsens with repetitive nerve stimulation (fatigability) as acetylcholine stores in the presynaptic nerve terminal are depleted. Thymic abnormalities (eg, thymoma, thymic hyperplasia) are common and may appear as an anterior mediastinal mass on imaging. Neuromuscular blocking agents are used to induce skeletal muscle relaxation during surgery by competing with acetylcholine at the postsynaptic membrane. They can be subclassified by their effects: Nondepolarizing agents (eg, vecuronium, rocuronium) are competitive antagonists of nicotinic acetylcholine receptors. Due to the low number of functioning receptors, patients with MG are extremely sensitive to these agents; very small doses can induce paralysis with loss of airway patency. Depolarizing agents (eg, succinylcholine) are competitive agonists of nicotinic acetylcholine receptors; persistent depolarization leads to desensitization of the motor end plate. Due to the low number of functioning receptors, patients with MG are often resistant to these agents; extremely large doses are required to induce muscle relaxation. (Choices A and B) Isoflurane is a general anesthetic (acts centrally on multiple receptors) that can cause malignant hyperthermia in those with genetic predispositions. Midazolam is a benzodiazepine (acts on the GABAA receptor) to produce sedation. These medications are not contraindicated in MG. (Choice C) The mainstay of symptomatic treatment for MG is acetylcholinesterase inhibitors (eg, pyridostigmine, neostigmine); these agents inhibit the degradation of acetylcholine in the synapse, thereby increasing the amount available to stimulate muscle contractions. (Choice D) Atropine is a muscarinic acetylcholine receptor antagonist often used as premedication before general anesthesia to decrease salivary and bronchial secretions (helps maintain airway patency) and prevent myocardial depression (minimizes risk of hypotension); it has no effect on nicotinic acetylcholine receptors in the neuromuscular junctions. Educational objective: Myasthenia gravis (MG) is caused by autoantibodies against postsynaptic nicotinic acetylcholine receptors, leading to fewer functional receptors and fatigable muscle weakness. Nondepolarizing neuromuscular blocking agents (eg, vecuronium) are competitive antagonists of nicotinic receptors; due to the depletion of receptors, patients with MG are extremely sensitive to these agents.

A 62-year-old woman is brought to the emergency department due to progressive weakness and dyspnea. The patient is visiting her niece, who says the patient forgot to bring her regular medications and has not been taking them for the past week. The patient began feeling weak and fatigued 3 days ago and has had shortness of breath since yesterday. Temperature is 37 C (98.6 F), blood pressure is 122/88 mm Hg, pulse is 90/min, and respirations are 24/min and shallow. On physical examination, the patient appears dyspneic with drooping at the eyelids and corners of the mouth. Her speech has a nasal quality. Arterial blood gas obtained on room air shows pH 7.32; PaCO2 is 52 mm Hg, PaO2 is 72 mm Hg, and HCO3 is 26 mEq/L. Bedside spirometry demonstrates decreased forced vital capacity. A pathologic process involving which of the following structures is the most likely cause of this patient's shortness of breath? Brain stem respiratory center Muscles of respiration Pulmonary arteries Terminal airways and alveoli Upper larger airways

This patient has hypercapnic and hypoxic respiratory failure (low pH, high CO2, low O2) indicating global hypoventilation. In association with the ptosis, bulbar weakness, and low forced vital capacity, this presentation suggests myasthenic crisis (severe weakness and respiratory depression due to an exacerbation of myasthenia gravis [MG]). MG is characterized by autoantibodies against nicotinic acetylcholine receptors on the postsynaptic membrane of the neuromuscular junction, resulting in receptor degradation. This reduces the sensitivity of the postsynaptic membrane to acetylcholine stimulation, leading to reduced muscular response despite normal acetylcholine release. Muscle weakness worsens with repetition as acetylcholine stores within the presynaptic nerve terminal become progressively depleted. Patients with MG typically have extraocular (eg, ptosis, diplopia), bulbar (eg, dysphonia, difficulty chewing), and facial (eg, myasthenic snarl) weakness. In addition, neck and proximal muscle weakness may occur, and in severe cases the respiratory muscles may be affected, leading to respiratory failure (as in this patient). Acetylcholinesterase inhibitors (eg, pyridostigmine, neostigmine) are used for symptomatic treatment, and withdrawal can trigger a myasthenic crisis. (Choice A) Impairment of the respiratory control centers in the brain stem causes hypoventilation during sleep (Ondine curse); however, voluntary breathing is unaffected. (Choice C) Pulmonary embolism can result in dyspnea and gas exchange abnormalities (eg, hypoxia). However, significant muscular weakness (eg, ptosis, nasal speech, decreased vital capacity) would be unexpected. (Choices D and E) Patients with diseases involving the terminal airways and alveoli (eg, pulmonary fibrosis, emphysema) and large airways (eg, asthma, chronic bronchitis) can present with respiratory failure subsequent to muscle fatigue. However, these diseases are not associated with non-respiratory muscle weakness (eg, dysphonia, bulbar weakness). Educational objective: Myasthenia gravis is characterized by autoantibodies against postsynaptic nicotinic acetylcholine receptors at the neuromuscular junction, which result in receptor degradation. Patients typically have extraocular, bulbar, and facial weakness that worsens with activity. In severe cases, the respiratory muscles may be affected, leading to hypoventilation and respiratory failure.

A 15-year-old boy is brought to the clinic due to poor exercise endurance. He recently began weight lifting with friends but has had difficulty performing the exercises. The patient states that his arms "feel like jelly after just a few repetitions." He also has severe muscle cramping and has noticed occasional urine discoloration after training sessions. Further evaluation reveals that the patient's exercise tolerance can be greatly improved by drinking an oral glucose solution before beginning a strenuous activity. Vital signs are normal, and examination is unremarkable. This patient is most likely deficient in an enzyme that catalyzes which of the following conversions? ID=1032 A B C D E

This patient most likely has McArdle disease (glycogen storage disease type V). This condition is caused by a deficiency of myophosphorylase, an isoenzyme of glycogen phosphorylase present in muscle tissue. Deficiency leads to decreased breakdown of glycogen during exercise, resulting in poor exercise tolerance, muscle cramps, and rhabdomyolysis (eg, red urine). The prognosis is generally good, and symptoms can be improved by consuming simple sugars before beginning physical activity. During glycogenolysis, glycogen phosphorylase shortens glycogen chains by cleaving 1,4-alpha-glycosidic linkages between glucose residues, liberating glucose-1-phosphate in the process. This occurs until 4 residues remain before a branch point (the limit dextrin). At this point, the debranching enzyme performs 2 enzymatic functions: Glucosyltransferase cleaves the 3 outer glucose residues of the 4 that are left by glycogen phosphorylase and transfers them to a nearby branch The enzyme alpha-1,6-glucosidase removes the single remaining branch residue, producing free glucose and a linear glycogen chain that can be further shortened by glycogen phosphorylase (Choice B) Glucose-1-phosphate generated by glycogenolysis is converted by phosphoglucomutase to glucose-6-phosphate. Unlike myophosphorylase, phosphoglucomutase is present in both skeletal muscle and liver. Phosphoglucomutase deficiency is extremely rare, presenting with muscle weakness and fasting hypoglycemia. (Choice C) Glucose-6-phosphate is converted to 6-phosphogluconate through a series of reactions by glucose-6-phosphate dehydrogenase and 6-phosphogluconolactonase in the pentose phosphate pathway. This pathway maintains adequate levels of reduced glutathione, which is needed to protect from oxidative injury (eg, prevent red cell hemolysis). (Choice D) Glucose-6-phosphate is converted to fructose-6-phosphate by glucose-6-phosphate isomerase during glycolysis. A deficiency here would disrupt glycolysis, affecting cells that rely on it as their energy source (eg, red blood cells). Patients have chronic hemolytic anemia. (Choice E) Within the liver and kidney, glucose-6-phosphatase converts glucose-6-phosphate to glucose to help maintain blood glucose levels during fasting. A deficiency here leads to glycogen storage disease type I (von Gierke disease), presenting in infancy with hypoglycemia. Educational objective: Myophosphorylase deficiency (McArdle disease, or glycogen storage disease type V) causes failure of muscle glycogenolysis, resulting in decreased exercise tolerance, muscle pain, cramping, and myoglobinuria shortly after initiating physical activity.

A 19-year-old woman is evaluated for new onset generalized tonic-clonic seizures. For the past several years, she has also been having erratic jerks of her arms and legs, and intermittent muscle weakness. The girl has multiple family members with similar symptoms. Neurological examination reveals decreased sensation in the lower extremities and a broad-based gait. Skeletal muscle biopsy shows ragged, red-appearing muscle fibers. Further analysis reveals that the patient's symptoms are due to a mutation affecting extranuclear DNA. Which of the following pedigrees is most likely to represent this patient's family history? (The arrow points to the patient.) ID=11914

This patient most likely has a form of mitochondrial myopathy. Without properly functioning mitochondria, cells are unable to use oxidative phosphorylation to efficiently produce adequate levels of ATP. Organ systems such as the brain and skeletal muscle will be affected first due to their high metabolic demand relative to other tissues. Affected patients often present with myopathy (eg, muscle weakness, myalgia), lactic acidosis due to impaired aerobic glycolysis, and nervous system dysfunction (eg, neuropathy, seizures). Muscle biopsy classically shows ragged red fibers. Mitochondria are unique organelles because they contain their own DNA, known as mtDNA. Offspring inherit mtDNA in a maternal fashion with no paternal contribution (maternal inheritance). Only affected females transmit abnormal mitochondria to offspring; transmission never occurs through males (even if they are affected). (Choice A) Autosomal dominant disorders affect 50% of all children (males and females) born to one affected parent. The disease will appear in consecutive generations, and father-to-son transmission can occur. (Choice B) Autosomal recessive conditions affect about 25% of all children (males and females) with two carrier parents. Offspring of a single affected parent will be carriers for the disorder. As a result, the disease can skip generations, but consanguineous families will show increased incidence. (Choice C) In X-linked dominant disorders, all female children of affected males will have the condition, but both female and male children of an affected female have a 50% chance of being affected. There is no father-to-son transmission. (Choice D) In X-linked recessive conditions, male offspring of a carrier female have a 50% chance of being affected, whereas female offspring have a 50% chance of being carriers. Female children of an affected father are obligate carriers. The disease can skip generations, and there is no father-to-son transmission. Educational objective: Mitochondrial dysfunction frequently presents with myopathy, nervous system dysfunction, lactic acidosis, and ragged red fibers on muscle biopsy. Mitochondrial myopathies due to mtDNA mutations are inherited solely in a maternal fashion (ie, maternal inheritance). Therefore, transmission occurs only through affected females and never through males.

A 30-year-old man is admitted to the hospital due to seizures. The repeated, prolonged tonic-clonic seizures were terminated with intravenous lorazepam in the emergency department. Medical history is significant for amphetamine abuse. The patient develops decreased urine output 24 hours after hospital admission. Temperature is 37.1 C (98.8 F), blood pressure is 140/90 mm Hg, pulse is 88/min, and respirations are 18/min. Examination shows bibasilar lung crackles and mild edema of the lower extremities. Laboratory results are as follows: Serum chemistry Blood urea nitrogen 40 mg/dL Creatinine 4.2 mg/dL Potassium 6.4 mEq/L Urinalysis Protein 2+ Blood 3+ White blood cells negative Red blood cells negative Which of the following is the most likely cause of this patient's kidney injury? Glomerular injury due to immune complexes Inflammatory reaction of the tubular interstitium Renal infarction due to arterial obstruction Tubular injury due to light-chain deposition Tubular injury due to released hemoglobin Tubular injury due to released myoglobin

This patient with acute kidney injury, hyperkalemia, and urinalysis with 3+ blood but no red blood cells has rhabdomyolysis, likely induced by his prolonged seizure. Rhabdomyolysis is characterized by myocyte injury with the release of intracellular muscle contents (ie, myoglobin, electrolytes) into the circulation. It is common in crush injuries, prolonged muscle activity (eg, seizure), or drug use. Positive blood on urine dipstick (a reaction that detects the heme pigment in both hemoglobin and myoglobin) in the absence of red blood cells on microscopic urinalysis suggests myoglobinuria. Renal injury in rhabdomyolysis results from myoglobin filtration and degradation within the glomeruli. Heme pigment is released, which causes acute tubular necrosis by direct cytotoxicity and renal vasoconstriction. Hyperkalemia, hyperphosphatemia, and hyperuricemia also occur due to myocyte lysis. (Choice A) Immune complex-mediated glomerular injury is seen in a variety of diseases (eg, IgA nephropathy, poststreptococcal glomerulonephritis), but these diseases do not cause myoglobinuria. Urinary cast formation or severe proteinuria are more common manifestations. (Choice B) Acute interstitial nephritis, an inflammatory reaction of the tubular interstitium, is typically associated with a medication exposure (eg, antibiotics) and presents with some combination of fever, eosinophilia, and rash. Urinalysis shows white blood cell casts, not myoglobinuria. (Choice C) Renal infarctions can be due to thromboembolic or atheroembolic disease and typically occur in older patients with established atherosclerosis or hypercoagulability. Hematuria with red cells would be expected on urinalysis. (Choice D) Multiple myeloma causes renal injury due to light chain-complex deposition in the renal tubules. This malignancy occurs in older patients and presents with hypercalcemia, anemia, and bone pain. It causes proteinuria (ie, Bence Jones protein), not myoglobinuria. (Choice E) Tubular injury due to released hemoglobin can also cause a heme pigment-induced kidney injury and may occur with hemolytic diseases (eg, paroxysmal nocturnal hemoglobinuria) or incompatible blood transfusion. However, renal failure after a prolonged seizure is more suggestive of rhabdomyolysis. Educational objective: Rhabdomyolysis is characterized by the release of intracellular muscle contents (eg, myoglobin, electrolytes) due to myocyte injury; it is common with crush injuries, seizures, or drug use (eg, statins). Heme pigment (released from myoglobin after degradation in the kidney) is toxic to tubular cells and can cause acute tubular necrosis. Positive blood on urine dipstick in the absence of red blood cells on microscopic urinalysis suggests myoglobinuria.

A 62-year-old woman has double vision, especially when reading at bedtime. The problem first started several months ago and was intermittent but has worsened over the last 2 weeks and now bothers her constantly. The patient has also developed difficulty chewing and swallowing foods. She usually feels better in the mornings. She has no extremity weakness or sensory symptoms. The patient has no other medical problems and takes no medications. She does not use tobacco, alcohol, or illicit drugs. Vital signs are normal. BMI is 27 kg/m2. Examination shows bilateral ptosis; there is weakness of adduction of the right eye and abduction of the left eye. Muscle strength is 4/5 in proximal muscles in the upper and lower extremities. Deep tendon reflexes are 2+ throughout. Item 1 of 2 Which of the following changes involving the postsynaptic muscle cell most likely explains this patient's symptoms? Impaired excitation-contraction coupling Prolonged absolute refractory period Reduced action potential amplitude Reduced action potential propagation velocity Reduced amplitude of motor end plate potential

This patient with progressive diplopia and weakness with mastication (both worse at night), along with ptosis and proximal muscle weakness on examination, likely has myasthenia gravis (MG). This disease is characterized by: Fluctuating weakness of certain voluntary muscles, particularly those innervated by motor nuclei of the brainstem (eg, ocular and bulbar muscles) Progressive weakening over the course of the day or after exercise (fatigability) Decreased compound muscle action potential amplitude (the summed action potentials of all the muscle fibers in a motor unit) on electromyography with repeated excitation Rapid restoration of strength of affected muscles with rest Improvement in strength following the administration of cholinesterase inhibitors MG is most commonly caused by autoantibodies against postsynaptic nicotinic acetylcholine receptors. Binding of antibody to these receptors results in blockade of the receptor's active site, receptor internalization and degradation, and damage to the motor end plate due to complement fixation. Overall, this leads to decreased numbers of functional acetylcholine receptors at the neuromuscular junction. The decrease in the number of available cation channels reduces the end plate potential following acetylcholine release. Because the threshold potential is not reached, the muscle cells do not depolarize. Synaptic concentrations of acetylcholine are unaffected, unlike in botulism or Lambert-Eaton syndrome. (Choice A) Excitation-contraction coupling refers to the process whereby an action potential within the muscle cell causes contraction of the muscle. In skeletal muscle, this process proceeds by the release of calcium into the cytoplasm, which subsequently binds troponin C and induces a conformational change allowing actin and myosin to bind. Impaired coupling implies that an action potential occurs but the skeletal muscle is unable to contract (eg, impaired actin and myosin binding), whereas MG is caused by a failure to achieve the threshold for an action potential to occur. (Choice B) The absolute refractory period for muscles in patients with MG is normal. (Choice C) The action potential amplitude of the postsynaptic muscle cell is determined by the properties of the cell membrane and is not affected by the quality of the stimulus at the muscle end plate. This is not the same as compound muscle action potential, which decreases with repeated excitation in patients with MG due to depolarization of fewer myofibers. (Choice D) Once the threshold voltage is exceeded at the neuromuscular end plate, an action potential propagates at a velocity determined by the particular properties of the muscle cell membrane. In MG, the muscle cell membrane is not affected outside of the neuromuscular junction. Educational objective: Myasthenia gravis is an autoimmune disease that causes a decrease in the number of functional acetylcholine receptors within the neuromuscular junction. This reduces the number of postsynaptic cation channels that can open in response to acetylcholine, which reduces the amplitude of motor end plate potential and prevents muscle fiber depolarization.

A 54-year-old previously healthy man comes to the office due to progressive fatigue and weakness for the past 2 months. His weakness predominantly involves the lower extremities, causing him to have difficulty rising from a chair. The patient also has a persistent cough and unintentional 7-kg (15.4-lb) weight loss during the same period. He has a 30-pack-year smoking history and drinks alcohol occasionally. Physical examination shows bilaterally decreased strength in the proximal muscles of the lower extremities with normal bulk and tone. Quadriceps tendon reflexes are diminished bilaterally. Chest x-ray reveals a hilar mass, biopsy of which demonstrates small cells that stain positive for chromogranin and synaptophysin. Which of the following is the most likely cause of this patient's muscle weakness? Antibodies against voltage-gated calcium channels Antibodies against nicotinic acetylcholine receptors Immune-mediated, segmental axonal demyelination Neoplastic deposits in the spinal cord epidural space Ubiquitin-proteasome-mediated muscle protein degradation

This patient with small cell lung cancer (small cells with neuroendocrine differentiation) has progressive proximal muscle weakness and diminished reflexes in the lower extremities, suggestive of paraneoplastic Lambert-Eaton myasthenic syndrome (LEMS). LEMS is an autoimmune disorder characterized by antibodies against presynaptic voltage-gated calcium channels at the neuromuscular junction. Normally, these calcium channels open in response to a presynaptic action potential, leading to an influx of calcium ions that cause synaptic vesicle fusion and acetylcholine release. LEMS-associated autoantibodies inhibit calcium influx, resulting in reduced acetylcholine release and failure to provoke muscular contractions. Muscular weakness in LEMS is progressive and predominantly involves the proximal muscles; it typically affects the lower limbs (eg, difficulty rising from a chair or climbing stairs) before the upper limbs (eg, raising arms above the head). Deep tendon reflexes are typically diminished or absent. Both strength and reflexes improve transiently with isometric muscular contractions in a phenomenon known as postexercise facilitation, likely due to a buildup of intracellular calcium with repetitive stimuli. Approximately 50% of patients have an underlying malignancy, most commonly small cell lung cancer. (Choice B) Autoantibodies against postsynaptic nicotinic acetylcholine receptors cause myasthenia gravis (MG), another immune-mediated neuromuscular disorder. MG typically involves the extraocular (eg, ptosis, diplopia) and bulbar (eg, dysphonia, dysphagia) muscles before affecting the limbs, and reflexes typically remain intact. (Choice C) Guillain-Barré syndrome occurs due to immune-mediated, segmental axonal demyelination; however, it typically occurs after an upper respiratory or gastrointestinal (eg, Campylobacter) infection. Rapidly progressive ascending paralysis (distal prior to proximal muscle involvement) is characteristic. (Choice D) Metastatic disease within the epidural space can cause spinal cord compression. Patients typically develop spastic paraparesis below the level of involvement. Hyperreflexia is common, and urine and stool retention may be seen (depending on the level of involvement). (Choice E) Protein degradation in cancer cachexia is associated with activation of the ubiquitin-proteasome system. Cancer cachexia causes muscle and fat loss, but weakness would be expected to affect proximal and distal muscles equally and reflexes are preserved. Educational objective: Lambert-Eaton myasthenic syndrome is characterized by antibodies against presynaptic voltage-gated calcium channels. Patients develop progressive proximal muscle weakness with reduced/absent reflexes that predominantly involves the lower extremities. The syndrome is often associated with malignancy, particularly small cell lung cancer.

A 34-year-old woman comes to the office with several months of vision problems, difficulty chewing, and trouble speaking. The symptoms fluctuate, but the patient has noticed that they are worse after a long day. She works as a telemarketer and has been putting in increasingly long hours to meet her quotas. Regarding her symptoms, she says, "It's probably just exhaustion; an ice pack to my face seems to help." During physical examination, the patient is asked to keep her eyes focused on a spot on the ceiling, and after 2 minutes, her vision becomes blurry. The remainder of the examination is normal. Further evaluation shows that the patient's symptoms are reversible on administration of an acetylcholinesterase inhibitor. The pathogenesis of this patient's disease is most similar to which of the following conditions? Atopic dermatitis Contact dermatitis Goodpasture syndrome Hypersensitivity pneumonitis Postinfectious glomerulonephritis

This patient's clinical findings of intermittent diplopia, difficulty chewing, and speech problems (especially at the end of the day) suggest a diagnosis of myasthenia gravis (MG). MG is caused by impaired functioning of nicotinic cholinergic receptors at the neuromuscular junction due to interference by IgG autoantibodies. The diagnosis can be confirmed with administration of an anticholinesterase agent (eg, edrophonium), which increases the amount of acetylcholine at the neuromuscular junction. Acetylcholinesterase activity decreases at lower temperatures, so application of an ice pack can also improve symptoms caused by weakness of superficial muscles (eg, ptosis). MG is a form of type II hypersensitivity, which is characterized by IgM and/or IgG autoantibodies that bind to cell surface antigens and/or extracellular matrix components. Among the available options, only Goodpasture syndrome is due to the production of autoantibodies (directed against the glomerular basement membrane). These antibodies cause inflammatory destruction of the basement membrane in the lung alveoli and renal glomeruli. (Choice A) Atopic dermatitis is a chronic, pruritic, eczematous skin condition that almost always begins in childhood. The pathogenicity involves several factors, including skin barrier abnormalities, defects in innate immunity, and a TH2-skewed immune response. (Choice B) The pathogenesis of contact dermatitis (type IV hypersensitivity reaction) involves cutaneous exposure to small foreign molecules (haptens) that bind to skin proteins. These proteins are subsequently processed by antigen-presenting Langerhans cells that transport the modified proteins to lymph nodes and promote the development of hapten-specific T cells. With continued hapten exposure, sensitized T cells produce inflammatory cytokines at the exposure site, causing the characteristic eczematous skin changes. (Choice D) Most patients with acute hypersensitivity pneumonitis have specific serum IgG antibodies that precipitate bacterial or fungal antigens found in inhaled organic dust particles. This causes interstitial alveolitis and bronchiolitis via immune complex and complement deposition in vessel walls (type III hypersensitivity mechanism). (Choice E) The pathogenesis of poststreptococcal glomerulonephritis involves granular deposition of immune complexes containing group A streptococcal antigens, IgG, and C3 in the glomerular basement membrane and mesangium. It is a type III, immune complex-mediated, hypersensitivity response. Educational objective: Myasthenia gravis results from an autoimmune type II (antibody-mediated) hypersensitivity reaction against skeletal myocyte surface acetylcholine receptors. Goodpasture syndrome similarly involves autoantibodies against basement membrane collagen in the renal glomeruli and lung alveoli.

A 21-year-old man is brought to the emergency department due to diffuse muscle aches and weakness. He has also noticed darkening of his urine. The patient recently joined the military and was participating in rigorous training exercises in hot weather earlier in the day. He has no significant medical conditions and takes no medications. Medical evaluation and laboratory testing performed prior to military enlistment showed no abnormalities. Temperature is 36.7 C (98 F), blood pressure is 100/60 mm Hg, pulse is 105/min, and respirations are 16/min. Physical examination shows dry mucous membranes and muscle tenderness over the bilateral thighs and calves. Laboratory results are as follows: Sodium 136 mEq/L Potassium 5.6 mEq/L Chloride 100 mEq/L Bicarbonate 18 mEq/L Blood urea nitrogen 30 mg/dL Creatinine 2.0 mg/dL Calcium 6.8 mg/dL Phosphorus 7.8 mg/dL Creatine kinase 22,000 U/L (normal: 30-170) Which of the following urine microscopy findings is most likely to be seen in this patient? Dysmorphic red blood cells Eosinophils Granular casts Isomorphic red blood cells Polymorphonuclear leukocytes Red blood cell casts

This patient's presentation is consistent with rhabdomyolysis, which is characterized by skeletal muscle necrosis and the release of intracellular breakdown products into the circulation. The condition is most commonly caused by trauma, sepsis, drugs/toxins (eg, statins, alcohol, cocaine), and overexertion (particularly in hot climates). Patients classically have myalgia and weakness (predominantly in the proximal muscles, lower back, and calves) and dark urine (due to myoglobinuria). Laboratory studies often show markedly elevated creatine kinase levels and acute kidney injury with electrolyte disturbances (eg, hyperkalemia, hyperphosphatemia, hypocalcemia, metabolic acidosis). Acute kidney injury in rhabdomyolysis occurs due to myoglobin degradation and heme pigment release. Heme pigment causes acute tubular necrosis (ATN) through direct cytotoxicity and renal vasoconstriction (ie, ischemia). In ATN, injured tubular epithelial cells slough off into the tubular lumen, forming granular, muddy brown casts. Heme pigment in myoglobin cross-reacts with the urine dipstick reagent that detects hemoglobin, leading to a false-positive result for blood in urine; however, microscopy shows no red blood cells (RBCs). (Choices A, D, and F) Dysmorphic RBCs and RBC casts are typically seen in patients with glomerulonephritis. Dysmorphic RBCs have abnormal shapes due to deformation as they pass through the glomerular basement membrane and osmotic stress in the renal tubules. Normal-appearing (isomorphic) RBCs are seen in nonglomerular sources of hematuria, such as nephrolithiasis or urinary tract malignancies. (Choice B) Urinary eosinophils are suggestive of acute interstitial nephritis (AIN), although they may be associated with other conditions (eg, kidney transplant rejection, pyelonephritis). AIN results from immune-mediated tubulointerstitial injury often caused by medications (eg, nonsteroidal anti-inflammatory drugs, penicillins). Patients classically have some combination of rash, fever, and eosinophilia. (Choice E) Polymorphonuclear leukocytes in the urine indicate inflammation, which most commonly occurs due to infection. Interstitial nephritis, renal tuberculosis, and gonorrhea/chlamydia urethritis should be considered in patients with negative urine cultures (sterile pyuria). Educational objective: Rhabdomyolysis usually presents with myalgia, proximal muscle weakness, and dark urine (myoglobinuria) in the setting of trauma, sepsis, or overexertion. Kidney injury occurs due to heme pigment-mediated tubular injury, leading to acute tubular necrosis. Urine microscopy typically reveals granular, muddy brown casts.

A 43-year-old man comes to the office with muscle weakness. His symptoms began 6 weeks ago and are worst in the hips and shoulders. The weakness has had a progressive course so that he is now having difficulty rising from chairs and combing his hair. The patient has had to reduce his regular exercise regimen due to weakness and has requested a modified work schedule for his job as a building maintenance manager. He has no history of spinal injury and no neck pain. Past medical history is insignificant, and the patient takes no medications. Muscle biopsy reveals major histocompatibility complex class I molecule overexpression on the sarcolemma with CD8+ lymphocyte infiltration. Which of the following is the most likely cause of this patient's condition? Diffuse systemic sclerosis Eaton-Lambert syndrome Myasthenia gravis Polyarteritis nodosa Polymyalgia rheumatica Polymyositis

This patient, a middle-aged man with progressive, symmetric proximal muscle weakness, has typical symptoms of polymyositis. The weakness may be painless or associated with diffuse myalgias. Muscle enzyme levels (eg, creatine kinase) are invariably elevated, and autoantibodies (eg, antinuclear antibodies, anti-Jo-1 antibodies) are present in most cases. Polymyositis is similar to dermatomyositis, although it lacks the typical skin findings; both may occur independently or as a paraneoplastic manifestation of an underlying malignancy. Polymyositis is an inflammatory myopathy triggered by unknown, possibly viral, antigens. It likely represents a cell-mediated immune response against myocytes. Increased expression of major histocompatibility complex class I antigens on the sarcolemma has been demonstrated and likely leads to presentation of autoantigens to CD8+ cytotoxic cells that subsequently initiate myocyte destruction. Muscle biopsy in polymyositis reveals inflammation, patchy necrosis, and regeneration and fibrosis of muscle fibers. Infiltration of the endomysium by macrophages and CD8+ lymphocytes is typically seen. (Choice A) Joint involvement in systemic sclerosis may cause arthralgias and contractures, but muscle weakness is not seen. (Choices B and C) Myasthenia gravis is caused by anti-acetylcholine receptor autoantibodies and is characterized by episodic weakness that initially affects the ocular/bulbar musculature. The Eaton-Lambert myasthenic syndrome is a paraneoplastic condition causing fatigable weakness in the extremities; it is caused by autoantibodies to presynaptic calcium channels. Light microscopy of muscle biopsy specimens is normal in these conditions. (Choice D) Polyarteritis nodosa is an uncommon systemic vasculitis that presents with intermittent episodes of abdominal pain, peripheral neuropathy, renal insufficiency, and severe hypertension. Biopsy is characterized by transmural inflammation of the arterial wall with fibrinoid necrosis. (Choice E) Polymyalgia rheumatica (PMR) causes myalgias of the shoulder and pelvic girdle muscles, often with systemic symptoms (eg, fever, weight loss). Weakness is not typical, and PMR occurs almost exclusively in patients age >50. Educational objective :Polymyositis causes symmetric proximal muscle weakness. Muscle biopsy reveals inflammation, necrosis, and regeneration of muscle fibers. Over-expression of major histocompatibility complex class I proteins on the sarcolemma leads to infiltration with CD8+ T lymphocytes and myocyte damage.

A 63-year-old man hospitalized for chronic obstructive pulmonary disease exacerbation develops hypercapnic respiratory failure. He is transferred to the intensive care unit, where he undergoes intubation after premedication with muscle relaxant X and an appropriate sedative. The patient is then started on mechanical ventilation; however, he remains apneic longer than expected. Anesthesiology is consulted, and the patient's neuromuscular blockade is assessed by electrically stimulating a peripheral nerve 4 times in quick succession and observing the muscular response. The procedure is repeated 30 minutes later and the results are shown below. ID=1212 Muscle relaxant X is most likely to be which of the following medications? Dantrolene Midazolam Pancuronium Succinylcholine Tubocurarine

Train-of-four (TOF) stimulation is used during anesthesia to assess the degree of paralysis induced by neuromuscular junction (NMJ)-blocking agents. A peripheral nerve is stimulated 4 times in quick succession and the muscular response is recorded. The height of each bar represents the strength of each twitch; higher bars indicate the activation of increasing numbers of individual muscle fibers (myocytes). When a nondepolarizing NMJ blocker (eg, vecuronium) is administered, competitive inhibition of postsynaptic acetylcholine receptors at the motor endplate prevents some of these fibers from activating, decreasing the strength of the twitch. TOF stimulation shows a progressive reduction in each of the 4 responses (fading pattern) as a result of less acetylcholine being released with each subsequent impulse (due to the additional effect of presynaptic acetylcholine receptor blockade). In contrast, depolarizing blockers (eg, succinylcholine) initially function by preventing repolarization of the motor endplate and show equal reduction of all 4 twitches during TOF stimulation (phase I blockade). The responses remain equal because the presynaptic acetylcholine receptor stimulation helps to mobilize presynaptic acetylcholine vesicles for release. Persistent exposure to succinylcholine results in eventual transition to phase II blockade as the acetylcholine receptors become desensitized and inactivated (ie, functionally similar to nondepolarizing blockade). Succinylcholine is commonly administered for rapid-sequence intubation due to its rapid onset (<1 minute). The duration of action is determined by its metabolism by plasma cholinesterase and is typically <10 minutes. However, some patients are homozygous for an atypical plasma cholinesterase, which breaks down succinylcholine more slowly. In these patients, the paralysis can last for hours and they must be maintained on mechanical ventilation until spontaneous respirations resume. (Choice A) Dantrolene relaxes skeletal muscle by reducing the release of Ca2+ from the sarcoplasmic reticulum. Dantrolene is used to treat malignant hyperthermia and neuroleptic malignant syndrome. It is not routinely used as a neuromuscular paralytic agent. (Choice B) Benzodiazepines are effective for sedation, but they have no direct action at the NMJ and do not provide sufficient muscle paralysis to facilitate intubation. (Choices C and E) Pancuronium and tubocurarine are nondepolarizing NMJ blockers. Unlike depolarizing NMJ drugs, these agents do not function in distinct phases and their TOF responses always display a fading pattern. Neostigmine administration reverses nondepolarizing NMJ blockade. Educational objective: Succinylcholine is a fast-acting, depolarizing neuromuscular blocking agent used for rapid-sequence intubation that causes equal reduction of all 4 twitches during train-of-four stimulation (phase I blockade). Prolonged administration of succinylcholine or use in patients with abnormal plasma cholinesterase activity causes transition to a phase II (nondepolarizing) block, seen as a progressive reduction in each of the 4 twitches.


Set pelajaran terkait

Unit 9 What is fastest animal in the world

View Set

Pelvic Girdle Bones (Pelvis and Hip)

View Set

DP-203: Data Engineering on Microsoft Azure

View Set

Mastering Microbiology Ch. 9 & 10

View Set

Chapter 6: Cartilage and Bones (Unit 2)

View Set